Preview (15 of 114 pages)

NCLEX RN ACTUAL EXAM TEST BANK OF REAL QUESTIONS &
ANSWERS
NCLEX 2023/2024 graded A
Part 3
QUESTION 401
What specific hormone must be present in serum or urine laboratory tests used to diagnose
pregnancy?
A. Human chorionic gonadotropin
B. Estrogen
C. fetoprotein
D. Sphingomyelin
Answer: A
Explanation:
(A) Human chorionic gonadotropin is the biochemical basis for pregnancy tests. It is
produced by the placenta to help maintain the corpus luteum. Its levels climb rapidly
following conception, peaking at about 8 weeks and then gradually decreasing to low levels
after 16 weeks.
(B) Estrogen does steadily rise throughout pregnancy, increasing to 30 times that of prepreg
Nancy levels. Although estrogen levels do change during pregnancy, it is not used as the main
hormone of evaluation in pregnancy tests.
(C) Fetoprotein is the major protein in the serum of the embryo. It is initially produced by the
yolk sac.
(D) Lecithin and sphingomyelin are two phospholipids of which fetal lung surfactant is
composed. Levels are evaluated to determine fetal lung maturity.
QUESTION 402
A 27-year-old primigravida at 32 week’s gestation has been diagnosed with complete placenta
previa. Conservative management including bed rest is the proper medical management. The
goal for fetal survival is based on fetal lung maturity. The test used to determine fetal lung
maturity is:
A. Dinitrophenylhydrazine
B. Metachromatic stain

C. Blood serum phenylalanine test
D. Lecithin-sphingomyelin ratio
Answer: D
Explanation:
(A) Dinitrophenylhydrazine is a laboratory test used to detect phenylketonuria, maple syrup
urine disease, and Lowe’s syndrome.
(B) Metachromatic stain is a laboratory test that may be used to diagnose Tay-Sachs and other
lipid diseases of the central nervous system.
(C) The blood serum phenylalanine test is diagnostic of phenylketonuria and can be used for
wide-scale screening.
(D) A lecithin-sphingomyelin ratio of at least 2:1 is indicative of fetal lung maturity, and
survival of the fetus is likely.
QUESTION 403
A client was exhibiting signs of mania and was recently started on lithium carbonate. She has
no known physical problems. A teaching plan for this client would include which of the
following?
A. Regular foods should be eaten, including those that contain salt, such as bacon, ham, V-8
juice, and tomato juice.
B. Restrict fluids to 1000 mL/day.
C. Restrict foods that contain salt or sodium.
D. Discontinue the medication if nausea occurs.
Answer: A
Explanation:
(A) This answer is correct. A balanced diet with adequate salt intake is necessary.
(B) This answer is incorrect. The client must drink six to eight full glasses of fluid per day
(2000–3000 mL/day).
(C) This answer is incorrect. The client should be instructed to avoid fluctuations of sodium
intake. Diet should be balanced, with an adequate salt intake.
(D) This answer is incorrect. Nausea is a frequent side effect that can be minimized with
administration of drug with meals or after eating food.

QUESTION 404
A client is hyperactive and not sleeping. She will not remain at the table during mealtime. She
is getting very limited calories and is using a lot of energy in her hyperactive state. The most
therapeutic nursing action is to:
A. Insist that she remain at the table and eat a balanced diet.
B. Order a high-calorie diet with supplements.
C. Provide nutritious finger foods several times a day.
D. Offer to go to the dining room with her and allow her to open the food and inspect what
she eats.
Answer: C
Explanation:
(A) The client is not able to sit for long periods. Forcing her to remain at the table will
increase her anxiety and cause her to become hostile.
(B) This action will not ensure that the client eats what is ordered. Dietary orders are not
within the nurse’s scope of practice.
(C) Providing finger foods increases the likelihood of eating for hyperactive persons. They
may be eating “on the run.”
(D) These clients are not suspicious of the food or insecure in moving about the unit alone.
QUESTION 405
A 52-year-old client who underwent an exploratory laparotomy for a bowel obstruction
begins to complain of hunger on the third postoperative day. His nasogastric (NG) tube was
removed this morning, and he has an IV of D5W with 0.45% normal saline running at 125
mL/hr. He asks when he can get rid of his IV and start eating. The nurse recognizes that he
will be able to begin taking oral fluids and nourishment when:
A. It is determined that he has no signs of wound infection
B. He is able to eat a full meal without evidence of nausea or vomiting
C. The nurse can detect bowel sounds in all four quadrants
D. His blood pressure returns to its preoperative baseline level or greater
Answer: C
Explanation:
(A) The absence of wound infection is related to his surgical wound and not to postoperative
GI functioning and return of peristalsis.

(B) Routine postoperative protocol involves detection of bowel sounds and return of
peristalsis before introduction of clear liquids, followed by progression of full liquids and a
regular diet versus a full regular meal first.
(C) Routine postoperative protocol for bowel obstruction is to assess for the return of bowel
sounds within 72 hours after major surgery, because that is when bowel sounds normally
return. If unable to detect bowel sounds, the surgeon should be notified immediately and have
the client remain NPO.
(D) Routine postoperative protocol for bowel obstruction and other major surgeries involves
frequent monitoring of vital signs in the immediate postoperative period (in recovery room)
and then every 4 hours, or more frequently if the client is unstable, on the nursing unit. This
includes assessing for signs of hypovolemic shock. Vital signs usually stabilize within the
first 24 hours postoperatively.
QUESTION 406
A 22-year-old client who is being seen in the clinic for a possible asthma attack stops
wheezing suddenly as the nurse is doing a lung assessment. Which one of the following
nursing interventions is most important?
A. Place the client in a supine position.
B. Draw a blood sample for arterial blood gases.
C. Start O2 at 4 L/min.
D. Establish a patent airway.
Answer: D
Explanation:
(A) During impending respiratory failure or asthmatic complications, the client is placed in
the high-Fowler position to facilitate comfort and promote optimal gas exchange.
(B) Arterial blood gases are monitored in the treatment of respiratory failure during an asthma
attack, but it is not an initial intervention.
(C) O2 therapy is used during an asthma attack, but it is not the initial intervention. The usual
prescribed amount is a cautiously low flow rate of 1–2 L/min.
(D) Wheezing is a characteristic clinical finding during an asthma attack. If wheezing
suddenly ceases, it usually indicates a complete airway obstruction and requires immediate
treatment for respiratory failure or arrest.

QUESTION 407
A 48-year-old client is being seen in her physician’s office for complaints of indigestion,
heartburn, right upper quadrant pain, and nausea of 4 days duration, especially after meals.
The nurse realizes that these symptoms may be associated with cholecystitis and therefore
would check for which specific sign during the abdominal assessment?
A. Cullen’s sign
B. Rebound tenderness
C. Murphy’s sign
D. Turner’s sign
Answer: C
Explanation:
(A) This sign is a faint blue discoloration around the umbilicus found in clients who have
hemorrhagic pancreatitis.
(B) This sign indicates areas of inflammation within the peritoneum, such as with
appendicitis. It is a deep palpation technique used on a nontender area of the abdomen, and
when the palpating hand is removed suddenly, the client experiences a sharp, stabbing pain at
an area of peritoneal inflammation.
(C) This sign is considered positive with acute cholecystitis when the client is unable to take a
deep breath while the right upper quadrant is being deeply palpated. The client will elicit a
sudden, sharp gasp, which means the gallbladder is acutely inflamed.
(D) This is a sign of acute hemorrhagic pancreatitis and manifests as a green or purple
discoloration in the flanks.
QUESTION 408
A 29-year-old client delivered her fifth child by the Lamaze method and developed a
postpartal hemorrhage in the recovery room. What are the initial symptoms of shock that she
may experience?
A. Marked elevation in blood pressure, respirations, and pulse
B. Decreased systolic pressure, cold skin, and anuria
C. Rapid pulse; narrowed pulse pressure; cool, moist skin
D. No urinary output, tachycardia, and restlessness
Answer: C
Explanation:

(A) Early shock does not exhibit the symptom of marked elevation in blood pressure. A
narrowing of the pulse pressure is indicative of early shock.
(B) Anuria is a clinical finding in late shock.
(C) All of these clinical findings are congruent with early shock.
(D) Absence of urinary output is a clinical finding in the late phase of shock.
QUESTION 409
A client is a depressed, 48-year-old salesman. A serious concern for the nurse working with
depressed clients is the potential of suicide. The time that suicide is most likely to occur is:
A. In the acutely depressed state
B. When the depression starts to lift
C. In the denial phase
D. During a manic episode
Answer: B
Explanation:
(A) The client may be too disorganized in the acute phase to make a workable plan.
(B) When the depression starts to lift, the client is able to make a workable plan.
(C) There usually is not a significant denial phase related to depression. Suicide occurs in a
state of despair and hopelessness.
(D) Suicide is uncommon in the manic state. In this state, clients do not feel hopeless, but
euphoric and overly confident.
QUESTION 410
The nurse assesses a client on the second postpartum day and finds a dark red discharge on
the peri pad. The stain appears to be about 5 inches long. Which of the following correctly
describes the character and amount of lochia?
A. Lochia alba, light
B. Lochia serosa, heavy
C. Lochia granulosa, heavy
D. Lochia rubra, moderate
Answer: D
Explanation:
(A) Lochia alba occurs approximately 10 days after birth and is yellow to white. A discharge
is classified as light when less than a 4-inch stain exists.

(B) Lochia serosa is pink to brown and occurs 3–4 days after delivery. A stain is classified as
heavy when a peri pad is saturated in 1 hour.
(C) Lochia granulosa is not a proper classification.
(D) Lochia rubra is red, consisting mainly of blood, debris, and bacteria, and lasts from the
time of delivery to 3–4 days afterward. A stain is classified as moderate when less than a 6inch stain exists.
QUESTION 411
A client sustained second- and third-degree burns to his face, neck, and upper chest. Which of
the following nursing diagnoses would be given the highest priority in the first 8 hours
postburn?
A. Fluid volume deficit secondary to alteration in skin integrity
B. Alteration in comfort secondary to alteration in skin integrity
C. Alteration in sensation secondary to third-degree burn
D. Alteration in airway integrity secondary to edema of neck and face, which in turn is
secondary to alteration in skin integrity
Answer: D
Explanation:
(A) Fluid deficit is a high priority not only during the first 8 hours postburn, but also during
the first 36 hours postburn.
(B) Alteration in comfort is a high priority during the entire length of the client’s
hospitalization and on discharge.
(C) Alteration in sensation is a high priority during the first 48–72 hours postburn. Lack of
sensation may be indicative of lack of circulation.
(D) Alteration in airway integrity is the highest priority for this client in the first 8 hours
postburn. Failure to continually assess this client’s airway status could result in poor
ventilation and oxygenation, in addition to an inability to intubate the client secondary to
excessive edema formation in the neck.
QUESTION 412
A 9-month-old infant was diagnosed with nonorganic failure to thrive. During her
hospitalization, primary nurses were assigned to initiate all infant feedings. The infant’s
parents question why they cannot feed their own child. Which of the following responses
would be most appropriate by the nurse?

A. By assigning the same nurses to the child, the nurses can begin to learn the infant’s cues
and feeding behaviors.
B. The same nurses will prevent parental fatigue and frustration.
C. The same nurses will prevent infant fatigue and frustration.
D. Primary nurses will ensure privacy.
Answer: A
Explanation:
(A) Consistent primary care nurses can better interpret infant cues and note feeding
behaviors.
(B) In nonorganic failure to thrive the parent-infant dyad has already experienced difficulties
in the relationship. These parents may already feel dissatisfied and frustrated. The primary
nurse would be unable to prevent this.
(C) Assigning a primary nurse does not ensure that infant fatigue and frustration will not
occur or can be prevented.
(D) Providing privacy does not ensure a change in feeding behavior.
QUESTION 413
A client was admitted to the hospital after falling in her home. At the time of admission, her
blood alcohol level was 0.27 mg%. Her family indicates that she has been drinking a fifth of
vodka a day for the past 9 months. She had her last drink 30 minutes prior to admission.
Alcohol withdrawal symptoms would most likely be exhibited by her:
A. Two to 4 hours after the last drink
B. Six to 8 hours after the last drink
C. Immediately on admission
D. Twenty-four hours after the last drink
Answer: B
Explanation:
(A) This answer is incorrect. Alcohol withdrawal usually begins approximately 6–8 hours
after the last drink.
(B) This answer is correct. It takes approximately 6–8 hours for metabolism of alcohol.
(C) This answer is incorrect.
The alcohol is still in the system, as indicated by the high blood alcohol level.
(D) This answer is incorrect. Symptoms of alcohol withdrawal usually begin within 6–8 hours
of the last drink.

QUESTION 414
A client’s renal calculi are identified as consisting of calcium phosphate. Which of the
following diets would be appropriate?
A. High calcium, low phosphorus
B. Low calcium, high phosphorus
C. Two-gram sodium diet
D. Low calcium and phosphorus, acid ash
Answer: D
Explanation:
(A) The stones consist of calcium and phosphorus; therefore, these minerals should be
avoided. A high-calcium diet is contraindicated.
(B) A high-phosphorus diet is contraindicated.
(C) A 2-g sodium diet is a cardiac diet.
(D) A low-calcium and phosphorus diet will reduce further calculi formation.
QUESTION 415
A 47-year-old client comes to the emergency department complaining of moderate flank,
abdominal, and testicular pain with nausea of 4 hours duration. After physical examination
and obtaining the client’s history, the physician suspects urethral obstruction by calculi. The
nurse realizes that the physician will order which one of the following diagnostic studies to
best confirm the diagnosis?
A. Cystoscopy
B. Kidneys, ureter, bladder, x-ray of abdomen
C. Intravenous pyelogram with excretory urogram
D. Ureter lithotomy
Answer: C
Explanation:
(A) Cystoscopy is an endoscopic procedure that uses an instrument (a cystoscope) to
visualize the internal bladder and ureter structures and to capture and remove an obstructing
stone.
(B) Kidney, ureter, bladder x-ray is used to outline gross structural changes in the kidneys,
ureter, and bladder and will determine the general location of a stone.

(C) An intravenous pyelogram with excretory urogram is used to visualize the kidneys,
kidney pelvis, ureters, and bladder. This procedure is used specifically to determine whether
urethral obstruction is partial or complete; it shows the exact location of the stone and dilation
of the ureter above the stone.
(D) Ureter lithotomy is a surgical procedure in which the ureter is incised and the stone is
manually removed because the stone is unable to pass through the ureter independently.
QUESTION 416
A male client was diagnosed 6 months ago with amyotrophic lateral sclerosis (ALS). The
progression of the disease has been aggressive. He is unable to maintain his personal hygiene
without assistance. Ambulation is most difficult, requiring him to use a wheelchair and rely
on assistance for mobility. He recently has become severely dysphasic. Nursing interventions
for dysphasia would be aimed toward prevention of:
A. Loss of ability to speak and communicate effectively
B. Aspiration and weight loss
C. Secondary infection resulting from poor oral hygiene
D. Drooling
Answer: B
Explanation:
(A) Loss of ability to speak is not dysphasia. Although the client may have difficulty
communicating, alternative measures can be developed to enhance communication. This goal,
while important, is of a lesser priority.
(B) Dysphasia is difficulty swallowing, which could result in aspiration of food and inability
to eat, causing weight loss.
(C) A secondary infection could result from poor oral hygiene, which could enhance the
client’s inability to eat, but this goal is of a lesser priority.
(D) Drooling normally occurs in clients with amyotrophic lateral sclerosis and may require
suctioning. Drooling, while aggravating for the client, does not pose an immediate danger.
QUESTION 417
A client had a ruptured abdominal aortic aneurysm that was repaired surgically. Her
postoperative recovery progressed without complications, and she is ready for discharge.
Client education in preparation for discharge began 7 days ago on her admission to the
nursing unit. Evaluation of nursing care related to client education is based on evaluation of

expected outcomes. Which statement made by the client would indicate that she is ready for
discharge?
A. “I will not drive but ride in the front seat of the car with a seat belt on for my first doctor’s
appointment.”
B. “When I bathe tomorrow morning, I will be very careful not to get soap on my incision.”
C. “I am allowed to exercise by walking for short periods.”
D. “Teach my husband about the diet. He’ll be doing all the cooking now.”
Answer: C
Explanation:
(A) Postoperatively, clients with major abdominal surgery are instructed to avoid driving,
riding in the front seat, and wearing seat belts because any sudden impact may injure a fresh
incision. She should ride in back seat without a seat belt.
(B) Clients should not sit in the tub and allow the incision to soak in water because this may
predispose the client to infection. A short, cool shower would be preferable. Allowing soap to
come in contact with the incision would not harm it and is frequently used as postoperative
wound care at home on discharge from the hospital.
(C) Activity instructions include: avoid sitting for long periods and get exercise by walking.
Lifting more than 5 lb of weight is also prohibited.
(D) The client must also learn her diet. Her husband cooking is probably a temporary measure
unless he did the cooking prior to her hospitalization statement such as this may indicate the
need for further exploration of feelings regarding her illness, dependence, and self-care
expectations.
QUESTION 418
A client has been diagnosed with congestive heart failure. His fluid intake and output are
strictly regulated. For lunch, he drank 8 oz of milk, 4 oz of tea, and 6 oz of coffee. His intake
would be recorded as:
A. 500 mL
B. 540 mL
C. 600 mL
D. 655 mL
Answer: B
Explanation:
(A, C, D) This answer is a miscalculation.

(B) 1 oz = 30 mL; therefore, 18 oz x.
QUESTION 419
A client takes warfarin (Coumadin) 15 mg po daily. To evaluate the medication’s
effectiveness, the nurse should monitor the:
A. prothrombin time (PT)
B. partial thromboplastin time (PTT)
C. PTT-C
D. Fibrin split products
Answer: A
Explanation:
(A) PT evaluates adequacy of extrinsic clotting pathway. Adequacy of warfarin therapy is
monitored by PT.
(B) PTT evaluates adequacy of intrinsic clotting pathway. Adequacy of heparin therapy is
monitored by PTT.
(C) There is no such laboratory test.
(D) Fibrin split products indicate fibrinolysis. This is a screening test for disseminated
intravascular coagulation. Heparin therapy may increase fibrin split products.
QUESTION 420
Following a bicycle accident, a 12-year-old client sustained a complete fracture of the left
femur. He was placed
A. In a long leg cast for immobilization.
B. In a splint with crutches for support.
C. In a short leg cast for limited mobility.
D. In skeletal traction to align the fracture.
Answer: D
Explanation:
(A) The client may have a knowledge deficit, but reducing the risk for knowledge deficit is
not a priority nursing diagnosis postoperatively.
(B) The client will have a Foley catheter for a day or two after surgery. Urinary retention is
usually not a problem once the Foley catheter is removed.
(C) A client having a cholecystectomy should not be physically impaired. In fact, the client is
encouraged to begin ambulating soon after surgery.

(D) Because of the location of the incision, the client having a cholecystectomy is reluctant to
breathe deeply and is at risk for developing pneumonia. These clients have to be reminded
and encouraged to take deep breaths.
QUESTION 421
A client has been admitted to the nursing unit with the diagnosis of severe anemia. She is
slightly short of breath, has episodes of dizziness, and complains her heart sometimes feels
like it will “beat out of her chest.” The physician has ordered her to receive 2 U of packed red
blood cells. The most important nursing action to be taken is:
A. Starting an 18-gauge IV infusion
B. Having the consent form on the chart
C. Administering the correct blood product to the correct client
D. Transfusing the blood in a 2-hour time frame
Answer: C
Explanation:
(A) An 18-gauge IV is an appropriate size for administering blood; however, client safety
demands that the right blood product must be administered.
(B) The consent form is legally necessary to be on the chart, but client safety is maintained by
giving the correct blood component to the correct client.
(C) Administering the correct blood product to the correct client will maintain physiological
safety and minimize transfusion reactions.
(D) The blood administration should take place over the ordered time frame designated by the
physician.
QUESTION 422
The physician has prescribed metoclopramide (Reglan). When assessing the client, the nurse
would expect to find which of the following responses?
A. Increase in gastric secretions
B. Increase in peristalsis
C. Disorientation
D. Drowsiness
Answer: B
Explanation:
(A) Metoclopramide does not stimulate gastric secretions.

(B) This response is expected with metoclopramide, in addition to increasing gastric
emptying.
(C) Disorientation is a symptom of metoclopramide overdose. The drug should be
discontinued.
(D) Drowsiness is a symptom of metoclopramide overdose and the drug should be
discontinued.
QUESTION 423
A client has received preoperative teaching for the vertical partial laryngectomy that he is
scheduled to have in
A. The need for a tracheostomy postoperatively.
B. The possibility of temporary voice changes after surgery.
C. The importance of maintaining proper hydration.
D. Increased fetal movements.
Answer: C
Explanation:
(A) This symptom would indicate a rupture of the membranes, which would be expected
during labor. There would be no cause for alarm if the fluid were clear.
(B) With uterine rupture and the risk of maternal shock secondary to blood loss, the most
likely sign would be hypotension indicating hypovolemic shock.
(C) In the event of a uterine rupture, an abdominal examination would likely reveal rigidity or
tenderness.
(D) The most likely finding would be a decrease in fetal movement related to fetal distress
due to impaired uteroplacental blood flow. Maintaining the client on her left side would help
to maximize uterine blood flow.
QUESTION 424
The physician prescribes amitriptyline (Elavil) for a client. What does the patient need to
know about this medication?
A. Prolonged use of this medication will result in extrapyramidal side effects.
B. When the medication is effective, he will experience no anxiety.
C. The medication should relieve his symptoms of depression.
D. Blood must be drawn weekly to test for toxicity.
Answer: C

Explanation:
(A) Phenothiazines cause extrapyramidal symptoms.
(B) No amount of medication can relieve all anxiety in all cases.
(C) The purpose of amitriptyline is to relieve the symptoms of depression because it is an
antidepressant. It increases the action of norepinephrine and serotonin on nerve cells.
(D) Periodic blood tests are done when lithium is prescribed.
QUESTION 425
A post-lung surgery client is placed on a chest tube drainage system. When explaining to the
family how the system works, the nurse states that the water-seal bottle of a three-bottle chest
drainage system serves which of the following purposes?
A. Collection bottle for drainage
B. Pressure regulator
C. Preventing accumulation of blood around the heart
D. Preventing air from entering the chest upon inspiration
Answer: D
Explanation:
(A) There is a separate collection bottle for drainage as part of a chest drainage system.
(B) In a three-bottle chest drainage system, one bottle serves only as a pressure regulator.
(C) Mediastinal chest tubes prevent accumulation of blood around the heart immediately
following heart surgery.
(D) The purpose of the water-seal bottle in any chest drainage setup is to allow air out of the
chest, but not back in. This negative pressure promotes lung expansion.
QUESTION 426
A behavioral modification program is recommended by the multidisciplinary team working
with a 15-year-old client with anorexia nervosa. A nursing plan of care based on this modality
would include:
A. Role playing the client’s eating behaviors
B. Restriction to the unit until she has gained 2 lb
C. Encouraging her to verbalize her feelings concerning food and food intake
D. Provision for a high-calorie, high-protein snack between meals
Answer: B
Explanation:

(A) This answer is incorrect. Role playing is based on learning but is not based on the
behavioral modification model.
(B) This answer is correct. The behavioral modification model is based on negative and
positive reinforcers to change behavior.
(C) This answer is incorrect. Verbal catharsis is not an intervention based on behavioral
modification.
(D) This answer is incorrect. Although an acceptable nursing intervention, it is not based on
behavioral modification.
QUESTION 427
A client is now pregnant for the second time. Her first child weighed 4536 g at delivery. The
client’s glucose tolerance test shows elevated blood sugar levels. Because she only shows
signs of diabetes when she is pregnant, she is classified as having:
A. Insulin-dependent diabetes
B. Type II diabetes mellitus
C. Type I diabetes mellitus
D. Gestational diabetes mellitus
Answer: D
Explanation:
(A) Insulin-dependent diabetes mellitus, also known as type I diabetes, usually appears before
the age of 30 years with an abrupt onset of symptoms requiring insulin for management. It is
not related to onset during pregnancy.
(B) Non-insulin–dependent diabetes (type II diabetes) usually appears in older adults. It has a
slow onset and progression of symptoms.
(C) This type of diabetes is the same as insulin-dependent diabetes.
(D) Gestational diabetes mellitus has its onset of symptoms during pregnancy and usually
disappears after delivery. These symptoms are usually mild and not life threatening, although
they are associated with increased fetal morbidity and other fetal complications.
QUESTION 428
A child becomes neutropenic and is placed on protective isolation. The purpose of protective
isolation is to:
A. Protect the child from infection
B. Provide the child with privacy

C. Protect the family from curious visitors
D. Isolate the child from other clients and the nursing staff
Answer: A
Explanation:
(A) The child no longer has normal white blood cells and is extremely susceptible to
infection.
(B) There are more appropriate ways to provide privacy, and there is no need to protect the
child from healthy visitors.
(C) Visitors and visiting hours may be at the client’s and/or family’s request without regard to
the isolation precaution.
(D) The child may have strong positive relationships with other clients or staff. As long as
proper precautions are observed, there is no reason to isolate her from them.
QUESTION 429
A client’s transfusion of packed red blood cells has been infusing for 2 hours. She is
complaining of a raised, itchy rash and shortness of breath. She is wheezing, anxious, and
very restless. The nurse knows these assessment findings are congruent with:
A. Hemolytic transfusion reaction
B. Febrile transfusion reaction
C. Circulatory overload
D. Allergic transfusion reaction
Answer: D
Explanation:
(A) A hemolytic transfusion reaction would be characterized by fever, chills, chest pain,
hypotension, and tachypnea.
(B) Fever, chills, and headaches are indicative of a febrile transfusion reaction.
(C) Circulatory overload is manifest by dyspnea, cough, and pulmonary crackles.
(D) Urticaria, pruritus, wheezing, and anxiety are indicative of an allergic transfusion
reaction.
QUESTION 430
A 75-year-old client is hospitalized with pneumonia caused by gram-positive bacteria. Which
one of the following best describes a gram-positive bacterial pneumonia?
A. Klebsiella pneumonia

B. Pneumococcal pneumonia
C. Legionella pneumophilapneumonia
D. Escherichia coli pneumonia
Answer: B
Explanation:
(A) Klebsiella pneumonia is caused by gram-negative bacteria.
(B) Pneumococcal pneumonia is caused by gram-positive bacteria.
(C) Legionella pneumophilapneumonia is a nonbacterial pneumonia.
(D) coli pneumonia is caused by gram-negative bacteria.
QUESTION 431
A 43-year-old client is admitted to the hospital with a diagnosis of peripheral vascular
disorder. She arrives in her room via stretcher and requires assistance to move to her bed. The
nurse notes that her left leg is cold to touch. She complains of having recently experienced
muscle spasms in that leg. To determine if these muscle spasms are indicative of intermittent
claudication, the nurse would begin her assessment with the following question:
A. “Would you describe the intensity, duration, and symptoms associated with your pain?”
B. “Do you experience swelling at the end of the day in the affected and unaffected leg?”
C. “Have you had any lesions of the affected leg that have been difficult to heal?”
D. “Do your muscle spasms occur following rest, walking, or exercising?”
Answer: D
Explanation:
(A) Describing pain is an important aspect of the assessment; however, assessing activity
preceding muscle spasms is equally important.
(B) Edema may occur with peripheral vascular disease, but it is not of particular importance
in assessing intermittent claudication.
(C) Lesions may be present with peripheral vascular disease, but they are not an indication of
intermittent claudication.
(D) With intermittent claudication, muscle spasms occur intermittently, mainly with walking
and after exercising. Rest may relieve muscle spasms.
QUESTION 432
A client has begun to exhibit signs of alcohol withdrawal. Her blood pressure has risen from
120/60 to 190/100, pulse is increased from 88 to 110 bpm, and she is irritable and agitated

and has gross motor tremors of the hands. The nurse notifies the doctor. The nurse can
anticipate that the doctor will order which of the following?
A. An opiate such as propoxyphene napsylate (Darvocet)
B. A benzodiazepine such as chlordiazepoxide (Librium)
C. A tricyclic antidepressant such as amitriptyline (Elavil)
D. A phenothiazine such as chlorpromazine (Thorazine)
Answer: B
Explanation:
(A) This answer is incorrect. Benzodiazepines are drugs of choice for alcohol withdrawal.
(B) This answer is correct. The drug has a sedative effect, is safe, and has an anticonvulsant
effect.
(C) This answer is incorrect. Amitriptyline is an antidepressant.
(D) This answer is incorrect. Chlorpromazine is most effective in psychotic disorders.
QUESTION 433
A 67-year-old man had a physical examination prior to beginning volunteer work at the
hospital. A routine chest x-ray demonstrated left ventricular hypertrophy. His blood pressure
was 180/110. He is 45 lb overweight. His diet is high in sodium and fat. He has a strong
family history of hypertension. The client is placed on antihypertensive medication; a lowsodium, low-fat diet; and an exercise regimen. On his next visit, compliance would best be
determined by:
A. A blood pressure reading of 130/70 with a 5-lb weight loss
B. No side effects from antihypertensive medication and an accurate pill count
C. No evidence of increased left ventricular hypertrophy on chest x-ray
D. Serum blood levels of the antihypertensive medication within therapeutic range
Answer: A
Explanation:
(A) A blood pressure within acceptable range best demonstrates compliance, but weight loss
cannot be accomplished without adherence to medication, diet, and exercise.
(B) Absence of side effects does not indicate compliance with medication. Pill counts can be
misleading because the client can alter pill counts prior to visit.
(C) Left ventricular hypertrophy is not an accurate measure of compliance because
hypertrophy frequently does not decrease even with pharmacological management.

(D) Therapeutic blood levels measure the drug level at the time of the test. There is no
indication of compliance several days before testing.
QUESTION 434
A 30-year-old client is exhibiting auditory hallucinations. In working with this client, the
nurse would be most effective if the nurse:
A. Encourages the client to discuss the voices
B. Attempts to direct the client’s attention to the here and now
C. Exhibits sincere interest in the delusional voices
D. Gives the medication as necessary for the acting-out behavior
Answer: B
Explanation:
(A) This answer is incorrect. Encouraging discussion of the voices will reinforce the delusion.
(B) This answer is correct. The nurse should appropriately present reality.
(C) This answer is incorrect. Showing interest would reinforce the delusional system.
(D) This answer is incorrect. The statement only indicates that the client is hearing voices. It
does not state that the client is acting out.
QUESTION 435
During an examination, the nurse notes that an infant has diaper rash on the convex surfaces
of his buttocks, inner thighs, and scrotum. Which of the following nursing interventions will
be most effective in resolving the condition?
A. Coating the inflamed areas with zinc oxide
B. Using talcum powder on the inflamed areas to promote drying
C. Removing the diaper entirely for extended periods of time
D. Cleaning the inflamed area thoroughly with disposable wet “wipes at each diaper change”
Answer: C
Explanation:
(A) Zinc oxide is not usually applied to inflamed areas because it contributes to sweat
retention.
(B) Talcum powder is of questionable benefit and poses a hazard of accidental inhalation.
(C) Removing the diaper and exposing the area to air and light facilitate drying and healing.
(D) Infants may be sensitive to one or more agents in the wet “wipes”. It is better to simply
clean with a wet cloth.

QUESTION 436
A 23-year-old college student seeks medical attention at the college infirmary for complaints
of severe fatigue.
Her skin is pale, and she reports exertional dyspnea. She is admitted to the hospital with
possible aplastic anemia. Laboratory values reflect anemia, and the client is prepared for a
bone marrow biopsy. She refuses to sign the biopsy consent and states, “Can’t you just get the
doctor to give me a transfusion and let me go.” This weekend begins spring break, and I have
plans to go to Florida. At this time the nurse’s greatest concern is that:
A. The client may contract an infection as a result of being exposed to large crowds at spring
break
B. The client does not grasp the full impact of her illness
C. The client may require transfusion before leaving for spring break
D. The causative agent be identified and treatment begun
Answer: B
Explanation:
(A) The client could contract an infection, but at this point it is not the most pertinent issue.
(B) The client’s statement indicates that she does not grasp the full impact of her illness.
Further client education must be given, along with allowing her to express her feelings
regarding her illness.
(C) The client may require a transfusion, but this is a temporary measure because the
causative agent has not been identified. Her feelings regarding her illness must be addressed
in order for care to continue.
(D) A bone marrow is done first to make a definitive diagnosis; then treatment may begin.
QUESTION 437
A client has a history of alcoholism. He is currently diagnosed with cirrhosis of the liver. The
nurse would expect him to be on which type of diet?
A. High protein and high calorie
B. High calorie and high carbohydrate
C. Low-fat 2-g sodium diet
D. High protein and high fat
Answer: B
Explanation:

(A) A high-protein diet is contraindicated in hepatic disease.
(B) High carbohydrates provide high-caloric content to prevent tissue catabolism.
(C) A low-fat 2-g sodium diet is a cardiac diet; however, a low-fat diet would be beneficial.
(D) A high-protein and high-fat diet is contraindicated in hepatic disease.
QUESTION 438
Prior to administering digoxin to a client with congestive heart failure, the nurse needs to
assess:
A. Respiratory rate for 1 minute
B. Radial pulse for 1 minute
C. Radial pulse for 2 minutes
D. Apical pulse for 1 minute
Answer: D
Explanation:
(A) Respiratory rate is not directly affected by digoxin therapy.
(B) A radial pulse is not as accurate as an apical pulse. Dysrhythmias may not be detected.
(C) A radial pulse is not as accurate as an apical pulse, regardless of assessment time.
(D) Apical pulse should be measured for 1-minute prior to digoxin administration. Digoxin
decreases the heart rate. Digoxin should be withheld if apical rates are 60 bpm or 120 bpm.
QUESTION 439
A client has been diagnosed with thrombophlebitis. She asks, “What is the most likely cause
of thrombophlebitis during my pregnancy?” The nurse explains:
A. Increased levels of the coagulation factors and a decrease in fibrinolysis
B. An inadequate production of platelets
C. An inadequate intake of folic acid during pregnancy
D. An increase in fibrinolysis and a decrease in coagulation factors
Answer: A
Explanation:
(A) During pregnancy, the potential for thromboses increases owing to the increased levels of
coagulation factors and a decrease in the breakdown of fibrin.
(B) An inadequate production of platelets would result in thrombocytopenia with resulting
signs and symptoms of bleeding such as petechiae, hematuria, or hematemesis.

(C) A deficiency of folic acid during pregnancy produces a megaloblastic anemia. It is usually
found in combination with iron deficiency.
(D) This combination would result in bleeding disorders because more fibrin would be broken
down and fewer clotting factors would be available.
QUESTION 440
A client had a renal transplant 3 months ago. He has suddenly developed graft tenderness, an
increased white blood cell count, and malaise. The client is experiencing which type of
rejection?
A. Acute
B. Chronic
C. Hyperacute
D. Hyperchronic
Answer: A
Explanation:
(A) The sudden development of fever, graft tenderness, increased white blood count, and
malaise are signs and symptoms of an acute rejection that commonly occurs at 3 months.
(B) Chronic rejection occurs slowly over a period of months to years and mimics chronic
renal failure.
(C) Hyperacute rejection occurs immediately after surgery up to 48 hours postoperatively.
(D) Hyperchronic rejection is not a type of rejection.
QUESTION 441
The nurse teaches a pregnant client that a high-risk symptom occurring at any time during
pregnancy that needs to be reported immediately to a healthcare provider is:
A. Constipation
B. Urinary frequency
C. Breast tenderness
D. Abdominal pain
Answer: D
Explanation:
(A) Constipation is a result of decreased peristalsis due to smooth muscle relaxation related to
changing progesterone levels that occur during pregnancy.

(B) Urinary frequency is a common result of the increasing size of the uterus and the resulting
pressure it places on the bladder.
(C) With the increased vascularity and hypertrophy of the mammary alveoli due to estrogen
and progesterone level changes, the breasts will increase in size and may become tender.
(D) Abdominal pain may be an indication of early spontaneous abortion, preterm delivery, or
a placental abruption.
QUESTION 442
An 8-year-old child is admitted to the hospital for surgery. She has had no previous
hospitalizations, and both she and her family appear anxious and fearful. It will be most
helpful for the nurse to:
A. Take the child to her room and calmly and matter-offactly begin to get her ready to go to
the operating room
B. Take time to orient the child and her family to the hospital and the forthcoming events
C. Explain that as soon as the child goes to the operating room she will have time to answer
any questions the family has
D. Tell the child and her family that there is nothing to worry about, that the operation will
not take long, and she will soon be as “good as new”
Answer: B
Explanation:
(A) This action does nothing to prepare the child and her family for what will happen or to
relieve their anxiety and fear.
(B) This action provides security by preparing the child and the family for what will happen
and will help to relieve fear and anxiety.
(C) This action does nothing to help prepare the child for what will happen and does not give
the parent’s permission to ask questions until later.
(D) This action provides possibly false reassurance and may prevent the child and/or the
family from asking pressing questions.
QUESTION 443
Diagnostic assessment findings for an infant with possible coarctation of the aorta would
include:
A. A third heart sound
B. A diastolic murmur

C. Pulse pressure difference between the upper extremities
D. Diminished or absent femoral pulses
Answer: D
Explanation:
(A) S1 and S2 in an infant with coarctation of the aorta are usually normal. S3 and S4 do not
exist with this diagnosis.
(B) Either no murmur will be heard or a systolic murmur from an associated cardiac defect
will be heard along the left upper sternal border. A diastolic murmur is not associated with
coarctation of the aorta.
(C) Pulse pressure differences of 20 mm Hg exist between the upper extremities and the
lower extremities. It is important to evaluate the upper and lower extremities with the
appropriate- sized cuffs.
(D) Femoral and pedal pulses will be diminished or absent in infants with coarctation of the
aorta.
QUESTION 444
Children often experience visual impairments. Refractive errors affect the child’s visual
activity. The main refractive error seen in children is myopia. The nurse explains to the
child’s parents that myopia may also be described as:
A. Cataracts
B. Farsightedness
C. Near-sightedness
D. Lazy eye
Answer: C
Explanation:
(A) Cataracts are not considered refractive errors. Cataracts cane described as opacity of the
lens.
(B) Hyperopiais the term for farsightedness. One can see objects at a distance more clearly
than close objects.
(C) Myopiais the term for near sightedness. Objects that are close in distance are more clearly
seen.
(D) Lazy eye refers to strabismus or misalignment of the eyes.

QUESTION 445
Which of the following serum laboratory values would the nurse monitor during gentamicin
therapy?
A. Creatinine
B. Sodium
C. Calcium
D. Potassium
Answer: A
Explanation:
(A) A common side effect of gentamicin is nephrotoxicity. The serum laboratory test that best
reflects kidney function is serum creatinine.
(B) Serum sodium has no relationship to gentamicin.
(C) Serum calcium has no relationship to gentamicin.
(D) Serum potassium has no relationship to gentamicin. If a client has impaired renal function
secondary to gentamicin administration, he or she may also have hyperkalemia as a secondary
disorder.
QUESTION 446
A 33-year-old client was brought into the emergency room unconscious, and it is determined
that surgery is needed. Informed consent must be obtained from his next of kin. The sequence
in which the next of kin would be asked for the consent would be:
A. Parent, spouse, adult child, sibling
B. Spouse, adult child, parent, sibling
C. Spouse, parent, sibling, adult child
D. Parent, spouse, sibling, adult child
Answer: B
Explanation:
(A) Spouse and adult child would be asked before a parent.
(B) The order of kin relationship for an adult, as determined from legal intestate succession, is
usually spouse, adult child, parent, sibling.
(C) Parent and sibling would be asked after adult child.
(D) Spouse and adult child would be asked before parent. Sibling would be asked last.

QUESTION 447
Based on your knowledge of genetic inheritance, which of these statements is true for
autosomal recessive genetic disorders?
A. Heterozygotes are affected.
B. The disorder is always carried on the X chromosome.
C. Only females are affected.
D. Two affected parents always have affected children.
Answer: D
Explanation:
(A) The term heterozygote refers to an individual with one normal and one mutant allele at a
given locus on a pair of homologous chromosomes. An individual who is heterozygous for
the abnormal gene does not manifest obvious symptoms.
(B) Disorders carried on either the X or Y sex chromosome are referred to as sex-linked
recessive.
(C) Either sex may be affected by autosomal recessive genetic disorders because the
responsible allele can be on any one of the 46 chromosomes.
(D) If both parents are affected by the disorder and are not just carriers, then all their children
would manifest the same disorder.
QUESTION 448
Prior to an amniocentesis, a fetal ultrasound is done in order to:
A. Evaluate fetal lung maturity
B. Evaluate the amount of amniotic fluid
C. Locate the position of the placenta and fetus
D. Ensure that the fetus is mature enough to perform the amniocentesis
Answer: C
Explanation:
(A) Amniocentesis can be performed to assess for lung maturity. Fetal ultrasound can be used
for gestational dating, although it does not separately determine lung maturity.
(B) Ultrasound can evaluate amniotic fluid volume, which may be used to determine
congenital anomalies.
(C) Amniocentesis involves removal of amniotic fluid for evaluation. The needle, inserted
through the abdomen, is guided by ultrasound to avoid needle injuries, and the test evaluates
the position of the placenta and the fetus.

(D) Amniocentesis can be performed as early as the 15th–17th week of pregnancy.
QUESTION 449
An expected response to sodium polystyrene sulfonate (Kayexalate) is:
A. Increase in serum magnesium
B. Increase in serum HCO3
C. Decrease in serum potassium
D. Decrease in serum calcium
Answer: C
Explanation:
(A) Sodium polystyrene sulfonate administration will not increase serum magnesium.
Hypermagnesemia is virtually unknown except for clients in renal failure.
(B) Sodium polystyrene sulfonate administration is not known to increase serum bicarbonate.
(C) Decrease in serum potassium, the expected response of sodium polystyrene sulfonate, is
secondary to the binding of this drug and potassium in the colon, and potassium is removed
through the feces.
(D) Serum calcium may actually increase with sodium polystyrene sulfonate administration,
especially if calcium chloride is administered concurrently with this drug.
QUESTION 450
A client who was started on antipsychotic medication 2 weeks ago is preparing for discharge
from the hospital. Compliance with the medication regimen is important despite the mild side
effects encountered. In order to increase the likelihood of medication compliance, the nurse
would:
A. Discuss the disease process and the importance of the medication in prevention of
symptoms.
B. Inform the client that additional side effects are to be expected and need not be reported.
C. Discuss the importance of getting blood drawn weekly to determine medication
therapeutics.
D. Inform the client to cease taking the medication when all psychotic symptoms have
cleared.
Answer: A
Explanation:

(A) This answer is correct. If the client is well informed about what reactions to expect from
her medication, she is more likely to follow the treatment regimen.
(B) This answer is incorrect. There are many side effects that are reversible by medication,
and these must be reported to the nurse or physician. There are also more severe side effects,
such as neuroleptic malignant syndrome, characterized by fever, tachycardia, and diaphoresis,
which can be life threatening.
(C) This answer is incorrect. There is no need for weekly blood tests if the drug regimen has
been followed properly.
(D) This answer is incorrect. The client should continue the medication until the physician
recommends any change in the drug regimen. Symptoms will usually reappear if medication
is discontinued.
QUESTION 451
A 37-year-old client has been taking antipsychotic medication for the past 10 days. The nurse
observes her walking with a shuffling gait and postural rigidity and notes a masklike
expression on her face. Which side effect is this client exhibiting?
A. Dystonia
B. Parkinsonism
C. Tardive dyskinesia
D. Akathesia
Answer: B
Explanation:
(A) This answer is incorrect. Dystonia refers to severe, painful muscle contractions.
(B) This answer is correct.
Parkinsonism commonly occurs approximately 1–2 weeks after initiation of antipsychotic
drug therapy. Traditional signs are masklike facies, postural rigidity, shuffling gait, and
resting tremor.
(C) This answer is incorrect. Tardive dyskinesia is characterized by involuntary muscle
movements of the face, jaw, and tongue.
(D) This answer is incorrect. Akathesia is motor restlessness.

QUESTION 452
A client is being discharged from the hospital tomorrow following a colon resection with a
left colostomy. The nurse knows that the client understands the discharge teaching about care
of her colostomy when she says:
A. “I know that I am not supposed to irrigate my colostomy.”
B. “My stool will be soft like paste.”
C. “My stoma should be red and slightly raised.”
D. “The skin around my stoma may become irritated from the enzymes in my stool.”
Answer: C
Explanation:
(A) A left colostomy indicates an ascending colon resection. This type of colostomy can be
irrigated.
(B) The stool from an ascending colon resection should be formed.
(C) The healthy stoma should be red and slightly raised. If it begins to turn dark or blue, the
client should see the physician immediately.
(D) The stool in the ascending colon does not usually have many enzymes in it. Stool from an
ileostomy has more enzymes and is more irritating to the skin.
QUESTION 453
A 25-year-old outpatient presents with a diagnosis of compulsive personality disorder. His
coworkers become annoyed with his rigid, perfectionistic manner and preoccupation with
trivial details and schedules. A nursing intervention appropriate for this client would include:
A. Encouraging him to engage in recreational activities
B. Avoiding discussion of his annoying behavior
C. Encouraging the client to set a time schedule and deadlines for himself
D. Contracting with him for the amount of time he will spend on the compulsive behaviors
Answer: D
Explanation:
(A) This answer is incorrect. The client will work hard at the activity instead of enjoying it.
(B) This answer is incorrect. The nurse should allow the client to discuss these thoughts,
within limits, not to avoid discussing them.
(C) This answer is incorrect. The compulsive client tends to control time to excess. It should
not be encouraged.

(D) This answer is correct. A contract with the client regarding the amount of time that will be
spent discussing the compulsive activities is appropriate. Time allotted should be gradually
decreased.
QUESTION 454
Pin care is a part of the care plan for a client who is in skeletal traction. When assessing the
site of pin insertion, which one of the following findings would the nurse know as an
indicator of normal wound healing?
A. Exudate
B. Crust
C. Edema
D. Erythema
Answer: B
Explanation:
(A) Exudate (moist, active drainage) is a clinical sign of wound infection.
(B) Crust (dry, scaly) is part of the normal stages of wound healing and should not be
removed from around the pin site. It usually sloughs off after the underlying tissue has healed.
(C) Edema (swelling) is a clinical sign of wound infection.
(D) Erythema (redness) is a clinical sign of wound infection.
QUESTION 455
A physician tells the nurse that he wants to orally intubate a client with a No. 8 endotracheal
tube. The finding of normal breath sounds on the right side of the chest and diminished,
distant breath sounds on the left side of the chest of a newly intubated client is probably due
to:
A. A left hemothorax
B. A right hemothorax
C. Intubation of the right mainstem bronchus
D. An inadequate mechanical ventilator
Answer: C
Explanation:
(A) Although a left hemothorax could cause diminished and distant breath sounds, it is
irrelevant to this situation.

(B) A right hemothorax will not cause diminished and distant breath sounds on the left side of
the chest.
(C) The right mainstem bronchus is most frequently intubated in error because the angle of
the right mainstem bronchus is very small as compared with that of the left mainstem
bronchus. Because ventilation is only occurring on the right side, the nurse would auscultate
diminished and distant breath sounds on the left.
(D) An inadequate mechanical ventilator has no relationship to this situation.
QUESTION 456
A 10-year-old client with a pin in the right femur is immobilized in traction. He is exhibiting
behavioral changes including restlessness, difficulty with problem solving, inability to
concentrate on activities, and monotony. Which of the following nursing implementations
would be most effective in helping him cope with immobility?
A. Providing him with books, challenging puzzles, and games as diversionary activities
B. Allowing him to do as much for himself as he is able, including learning to do pin-site care
under supervision
C. Having a volunteer come in to sit with the client and to read him stories
D. Stimulating rest and relaxation by gentle rubbing with lotion and changing the client’s
position frequently
Answer: B
Explanation:
(A) These activities could be frustrating for the client if he is having difficulty with problem
solving and concentration.
(B) Selfcare is usually well received by the child, and it is one of the most useful
interventions to help the child cope with immobility.
(C) This may be helpful to the client if he has no visitors, but it does little to help him develop
coping skills.
(D) This will help to prevent skin irritation or breakdown related to immobility but will not
help to prevent behavioral changes related to immobility.
QUESTION 457
A hyperactive client is experiencing flight of ideas. The most therapeutic activity for him
would be:
A. Doing crafts in occupational therapy

B. Working a 1000-piece puzzle
C. Playing bridge with three other clients
D. Playing basketball in the gym
Answer: D
Explanation:
(A) This activity requires motor skills and therefore would be difficult for a hyperactive
client.
(B) This activity would take too long, and the client would have difficulty concentrating
owing to a limited attention span.
(C) This client would not be able to concentrate enough to play card games. He would
respond to all the stimuli in the area, become distracted, and leave the table.
(D) This activity would allow the client to channel his energy in a positive way.
QUESTION 458
A male client was involved in a motor vehicle accident earlier in the day. The nurse caring for
him on evenings notices that on admission to the hospital, he lost a lot of blood and required
multiple blood transfusions. The nurse would anticipate which blood product would be
ordered when a large blood loss has occurred?
A. Whole blood
B. Platelets
C. Fresh frozen plasma
D. Packed red blood cells
Answer: A
Explanation:
(A) Whole blood is the transfusion component of choice when large volumes of blood need to
be replaced. Whole blood contains all blood components that are lost during active bleeding.
(B) Platelet therapy is indicated for thrombocytopenia if the client’s platelet count is below
15,000/mm3.
(C) Infusion of fresh frozen plasma is required when the prothrombin time and partial
thromboplastic time are prolonged.
(D) Packed red blood cells are transfused in instances of anemia with decreases in hematocrit
and hemoglobin.

QUESTION 459
A 9-month-old infant is being examined in the general pediatric clinic for a routine well-child
checkup. His immunizations are up to date, and his mother reports that he has had no
significant illnesses or injuries. Which of the following signs would lead the nurse to believe
that he has had a cerebral injury?
A. Hyperextension of the neck with evidence of pain on flexion
B. Holding the head to one side and pointing the chin toward the other side
C. Holding the head erect and in the midline when in a vertical position
D. Significant head lag when raised to a sitting position
Answer: D
Explanation:
(A) This position is indicative of a possible meningeal irritation or infection such as
meningitis.
(B) This position is seen most frequently in infants who have had an injury to the
sternocleidomastoid muscle.
(C) Most infants aged 4 months and older are able to maintain this position.
(D) Infants older than 6 months of age should not have significant head lag. This is a sign of
cerebral injury and should be referred for further evaluation.
QUESTION 460
The physician prescribes a medical regimen of isoniazid, rifampin, and vitamin B6 for a
tuberculosis client. The nurse instructs the client that B6 is given because it:
A. Increases activity of isoniazid
B. Increases activity of rifampin
C. Improves nutritional status
D. Reduces peripheral neuropathy
Answer: D
Explanation:
(A) Vitamin B6 does not enhance the activity of isoniazid.
(B) Vitamin B6 does not enhance the activity of rifampin.
(C) A vitamin alone does not improve nutritional status.
(D) Isoniazid leads to Vitamin B6deficiency, which is manifested as peripheral neuropathy.

QUESTION 461
A client is being discharged from the hospital today. The discharge teaching for care of her
colostomy included which of the following basic principles for protecting the skin around her
stoma:
A. Taping a pouch that is leaking
B. Cutting the skin barrier 112 inches larger than the stoma
C. Changing the pouch only when leakage occurs
D. Using a skin sealant under pouch adhesives
Answer: D
Explanation:
(A) When a pouch seal leaks, the pouch should be immediately changed, not taped. Stool held
against the skin can quickly result in severe irritation.
(B) The skin barrier should be cut only slightly larger than the stoma (one-half inch).
(C) The client should be taught to change pouches whenever possible before leakage occurs.
(D) When skin sealant is used under the tape, the outermost layer of the epidermis remains
intact. When no skin sealant is used, this layer is removed when the tape is removed.
QUESTION 462
In the coronary care unit, a client has developed multifocal premature ventricular
contractions. The nurse should anticipate the administration of:
A. Furosemide
B. Nitroglycerin
C. Lidocaine
D. Digoxin
Answer: C
Explanation:
(A) Furosemide is a loop diuretic.
(B) Nitroglycerin is a vasodilator.
(C) Lidocaine is the drug of choice to treat ectopic ventricular beats.
(D) Digoxin slows down the electrical impulses and increases ventricular contractions, but it
does not rapidly correct ventricular ectopy.

QUESTION 463
On the first postpartal day, a client tells the nurse that she has been changing her perineal pads
every 1/2 hour because they are saturated with bright red vaginal drainage. When palpating
the uterus, the nurse assesses that it is somewhat soft, 1 fingerbreadth above the umbilicus,
and midline. The nursing action to be taken is to:
A. Gently massage the uterus until firm, express any clots, and note the amount and character
of lochia
B. Catheterize the client and reassess the uterus
C. Begin IV fluids and administer oxytocic medication
D. Administer analgesics as ordered to relieve discomfort
Answer: A
Explanation:
(A) Gentle massage and expression of clots will let the fundus return to a state of firmness,
allowing the uterus to function as the “living ligature.”
(B) A distended bladder may promote uterine atony; however, after determining the bladder is
distended, the nurse would have the client void. Catheterization is only done if normal
bladder function has not returned.
(C) Oxytocic medications are ordered and administered if the uterus does not remain
contracted after gentle massage and determining if the bladder is empty.
(D) The client is not complaining of discomfort or pain; therefore, analgesics are not
necessary.
QUESTION 464
When providing dietary teaching to an individual who has diabetes mellitus, type II, the nurse
discusses the importance of consuming the recommended daily allowance of which of the
following electrolytes?
A. Potassium
B. Magnesium
C. Sodium
D. HCO3
Answer: B
Explanation:

(A) Potassium intake that meets the recommended daily allowance is important, especially in
clients who have a history of cardiac disease.
(B) Low levels of magnesium can cause an increase in resistance to insulin and can lead to
carbohydrate intolerance.
(C) Sodium is an important electrolyte for all clients but has no direct effect on diabetes
mellitus.
(D) Bicarbonate plays an important role in acid-base balance. It is equally necessary for
maintenance of all body functions.
QUESTION 465
A 52-year-old client is scheduled for a small-bowel resection in the morning. In conjunction
with other preoperative preparation, the nurse is teaching her diaphragmatic breathing
exercises. She will teach the client to:
A. Inhale slowly and deeply through the nose until the lungs are fully expanded, hold the
breath a couple of seconds, and then exhale slowly through the mouth. Repeat 2–3 more
times to complete the series every 1–2 hours while awake
B. Purse the lips and take quick, short breaths approximately 18–20 times/min
C. Take a large gulp of air into the mouth, hold it for 10–15 seconds, and then expel it through
the nose. Repeat 4–5 times to complete the series
D. Inhale as deeply as possible and then immediately exhale as deeply as possible at a rate of
approximately 20– 24 times/min
Answer: A
Explanation:
(A) This is the correct method of teaching diaphragmatic breathing, which allows full lung
expansion to increase oxygenation, prevent atelectasis, and move secretions up and out of the
lungs to decrease risk of pneumonia.
(B) Quick, short breaths do not allow for full lung expansion and movement of secretions up
and out of the lungs. Quick, short breaths may lead to O2 depletion, hyperventilation, and
hypoxia.
(C) Expelling breaths through the nose does not allow for full lung expansion and the use of
diaphragmatic muscles to assist in moving secretions up and out of the lungs.
(D) Inhaling and exhaling at a rate of 20–24 times/min does not allow time for full lung
expansion to increase oxygenation. This would most likely lead to O2 depletion and hypoxia.

QUESTION 466
The client tells the nurse, “I have pain in my left shoulder.” This is considered:
A. Evaluation process
B. Objective information
C. Subjective information
D. Complaining
Answer: C
Explanation:
(A) Evaluation process follows a nursing intervention.
(B) Objective information can be measured.
(C) Subjective information is provided by a person.
(D) Client is reporting a symptom that needs to be assessed.
QUESTION 467
A 10-month-old infant’s mother says that he takes fresh whole milk eagerly, but that when she
offered him baby foods at 6 months of age, he pushed them out of his mouth. Because he has
gained weight appropriately, she has quit trying to get him to eat other foods. The nurse’s
response is based on the knowledge that:
A. Milk intake should be limited to no more than four 8-oz bottles per day and should be
followed by iron- enriched cereal or other solid foods or juices
B. Milk is an excellent food and will meet his nutritional needs adequately until he is ready to
eat solid foods
C. It is acceptable to continue to give him whole milk and to delay giving solid foods as long
as he takes a vitamin supplement daily
D. He should be started on iron-enriched cereal, meat, vegetables, fruits, and juices prior to
bottle feeds. Milk intake should be limited to 1 qt/day
Answer: D
Explanation:
(A) If the infant is given the bottle first, he will be less likely to be hungry enough to eat the
solid foods.
(B) Milk is deficient in iron, vitamin C, zinc, and fluoride. It does not provide an adequate
diet.
(C) The vitamin supplement will help, but the infant needs an iron supplement.

(D) Giving the solid food when the infant is hungriest will increase the likelihood that he will
eat. The more solid food he takes, the less milk he will desire.
QUESTION 468
A 32-year-old mother of two was brought to the hospital by her husband. He reported that his
wife could no longer manage the house and children. She does not sleep and talks day and
night. She has purchased some very expensive clothes. The nurse noted that the client speaks
rapidly and changes the subject irrationally. This is an example of:
A. Flight of ideas
B. Delusions
C. Hallucinations
D. Echolalia
Answer: A
Explanation:
(A) Rapidly moving from one topic to another without following any logical sequence is
called flight of ideas.
(B) False beliefs are delusions.
(C) False sensory perceptions are hallucinations (“hearing voices”).
(D) Repeating words is called echolalia.
QUESTION 469
During his hospitalization, a 3-year-old child has become unusually aggressive in his play
activities. His parents report this change in behavior to the primary nurse. How could the
nurse explain the child’s change in behavior?
A. Deep-seated feelings of hostility
B. A lack of interest in socializing
C. Usual behavior for this child
D. A coping response
Answer: D
Explanation:
(A) Unusually aggressive behavior does not indicate a deep-seated problem.
(B) A lack of social interest results in poor participation in play activities with peers.
Aggression would not be an expected behavior.
(C) The aggressive behavior was newly developed and not a routine behavior.

(D) Play provides the child with opportunities for coping and adaptation. Aggression during
the play activities would indicate a coping response.
QUESTION 470
A 55-year-old woman entered the emergency room by ambulance. Her primary complaint is
chest pain. She is receiving O2 via nasal cannula at 2 L/min for dyspnea. Which of the
following findings in the client’s nursing assessment demand immediate nursing action?
A. Associated symptoms of indigestion and nausea
B. Restlessness and apprehensiveness
C. Inability to tolerate assessment session with the admitting nurse
D. History of hypertension treated with pharmacological therapy
Answer: B
Explanation:
(A) Indigestion or nausea may accompany angina or myocardial infarction, but they do not
indicate imminent danger for the client.
(B) Restlessness and apprehensiveness require immediate nursing action because they are
indicative of very low oxygenation of body tissues and are frequently the first indication of
impending cardiac or respiratory arrest.
(C) It is common for the cardiac client to experience fatigue and inability to physically
tolerate long assessment sessions.
(D) A history of hypertension requires no immediate nursing intervention. In the situation
described, the blood pressure is not given and therefore cannot be assumed to be elevated.
QUESTION 471
To facilitate maximum air exchange, the nurse should position the client in:
A. High Fowler
B. Orthopneic
C. Prone
D. Flat-supine
Answer: B
Explanation:
(A) The high Fowler position does increase air exchange, but not to the extent of orthopneic
position.
(B) The orthopneic position is a sitting position that allows maximum lung expansion.

(C) The prone position places pressure on diaphragm and does not promote maximum air
exchange.
(D) The flat-supine position places pressure on diaphragm by abdominal organs and does not
promote maximum air exchange.
QUESTION 472
A 26-year-old client is diagnosed with an astrocytoma, a benign brain tumor. From the nurse’s
knowledge of the central nervous system, the nurse knows that benign tumors:
A. Can be just as dangerous as malignant tumors
B. Grow more rapidly than malignant tumors
C. Do not warrant concern because they do not become malignant tumors
D. Can be removed surgically
Answer: A
Explanation:
(A) Both a benign and a malignant tumor can displace or destroy nearby structures or
increase intracranial pressure.
(B) Benign or malignant brain tumors grow at different rates depending on the type of tumor.
(C) Some benign tumors do become malignant tumors.
(D) Whether or not a tumor is operable depends on its location and the amount of damage its
removal will cause.
QUESTION 473
A 4-year-old child with a history of sickle cell anemia is admitted to the nursing unit with
dizziness, shortness of breath, and pallor. Nursing assessment findings reveal tenderness in
the abdomen. The child is most likely experiencing a/an:
A. Aplastic crisis
B. Vaso-occlusive crisis
C. Dactylitis crisis
D. Sequestration crisis
Answer: D
Explanation:
(A) Aplastic anemia is characterized by a lack of reticulocytes in the blood. Platelet and white
blood cell counts are usually not depressed. It is usually self-limiting, lasting 5–10 days.
(B) Vaso-occlusive crisis is the most common type of crisis in sickle cell anemia.

Sickled cells become clogged, leading to distal tissue hypoxia and infarction. Joints and
extremities are the most commonly affected areas.
(C) Dactylitis crisis, or “hand-foot syndrome”, causes symmetrical infarction of the bones in
the hands and feet, resulting in painful swelling in the soft tissues of the hands and feet.
(D) Sequestration crisis occurs as enormous volumes of blood pool within the spleen. The
spleen enlarges, causing tenderness. Signs of shock including pallor, tachypnea, and faintness
result, related to the deficient intravascular volume. This type of crisis is potentially fatal.
QUESTION 474
A client who has gout is most likely to form which type of renal calculi?
A. Struvite stones
B. Staghorn calculi
C. Uric acid stones
D. Calcium stones
Answer: C
Explanation:
(A) The presence of urinary tract infection is a factor in the formation of struvite stones.
(B) Staghorn calculi is the other name for struvite stones associated with urinary tract
infection.
(C) Clients who have gout form uric acid stones.
(D) Clients who have increased urinary excretion of calcium form calcium stones.
QUESTION 475
A school-age child with asthma is ready for discharge from the hospital. His physician has
written an order to continue the theophylline given in the hospital as an oral home
medication. Immediately prior to discharge, he complains of nausea and becomes irritable.
His vital signs were normal except for tachycardia. What first nursing actions would be
essential in this situation?
A. Hold the child’s discharge for 1 hour.
B. Notify the physician immediately.
C. Discharge the child as the physician ordered.
D. Administer an antiemetic as necessary.
Answer: B
Explanation:

(A) Holding the child’s discharge alone does not address the client’s problem.
(B) Nausea, tachycardia, and irritability are all symptoms of theophylline toxicity. The
physician should be notified immediately so that a serum theophylline level can be ordered.
Theophylline dose should be withheld until the physician is notified.
(C) The child must be evaluated for theophylline toxicity before any discharge.
(D) Cause of the nausea should be investigated before the administration of an antiemetic.
QUESTION 476
Nursing care for the parents of a child with a congenital heart defect would include:
A. Encouraging the parents not to tell the child about the seriousness of the congenital heart
defect, so the child will function as normally as possible
B. Acknowledging the fear and concern surrounding their child’s health and assisting the
parents through the grieving process as they mourn the loss of their fantasized healthy child
C. Identifying anger and resentment as destructive emotions that serve no purpose
D. Expressing to the parents after the corrective surgery has been completed successfully that
all their grief feelings will resolve
Answer: B
Explanation:
(A) It is important to discuss with parents the need to treat the child as they would any other
children, but they must be truthful and honest with the child about the heart defect. As the
child grows older, explanations can go into greater depth.
(B) Parents of children with congenital heart defects go through a grieving process over the
loss of their “healthy child”. The nurse needs to recognize these feelings and give the parents
a role in the child’s care when they are ready.
(C) Anger and resentment are normal feelings that must be dealt with appropriately.
(D) Parents may go through a second grieving process after the repair of the cardiac defect.
During this grieving period, they mourn the loss of the “defective child” who now may be
essentially “normal”.
QUESTION 477
When teaching a class of nursing students, the nurse asks why the embryonic period (weeks
4–8) of pregnancy is so critical.
A. Duplication of genetic information takes place.
B. Organogenesis occurs.

C. Subcutaneous fat builds up steadily.
D. Kidneys begin to secrete urine.
Answer: B
Explanation:
(A) Duplication of genetic material occurs during the PR embryonic period (weeks 1–3)
following conception. The exact duplication of genetic material is essential for cell
differentiation, growth, and biological maintenance of the organism.
(B) Weeks 4–8, known as the embryonic period, are the time organogenesis occurs and pose
the greatest potential for major congenital malformations. All major internal and external
organs and systems are formed.
(C) Subcutaneous fat does not develop until the latter weeks of gestation.
(D) Kidneys begin to secrete urine during the 13th–16th week.
QUESTION 478
Parents of young children often need anticipatory guidance from the nurse. Parents may have
little knowledge regarding growth and development. Which of the following toys and
activities would the nurse suggest as appropriate for a toddler?
A. Cutting, pasting, string beads, music, dolls
B. Mobiles, rattle, squeeze toys
C. Pull-toys, large ball, dolls, sand and water play, music
D. Simple card games, puzzles, bicycle, television
Answer: C
Explanation:
(A) These activities are suited for the preschool-age child (3–5 years old). The activities are
not safe for a toddler.
(B) Infants (0–1 year) like these toys.
(C) These activities provide the toddler (1–3 years old) with a variety of physical activities for
play.
(D) The toddler lacks the physical and cognitive abilities for these activities. The tasks are far
better suited for the school-age child.
QUESTION 479
A 5-year-old child was recently diagnosed as having acute lymphoid leukemia. She is
hospitalized for additional tests and to begin a course of chemotherapy designed to induce a

remission. She is scheduled to have a bone marrow aspiration tomorrow. She has had a bone
marrow test previously and is apprehensive about having another. Which of the following
interventions will be most effective in relieving her anxiety?
A. Explain what will take place and what she will see, feel, and hear.
B. Remind her that she has had this procedure before and that it is nothing to be afraid of.
C. Tell her not to worry about it, that it will be over soon and she can join her friends in the
playroom.
D. Give her a big hug and tell her that she is a big girl now and that she will do just fine.
Answer: A
Explanation:
(A) Even though the child has had the procedure before, she will probably need additional
explanations and emotional support.
(B) The fact that the child has had the procedure before and possibly found it painful or
uncomfortable may increase, not relieve, her stress.
(C) This intervention does nothing to reassure the child and may well prevent her from
expressing her feelings.
(D) This does not prepare the child for the test and burdens her with the expectation that she
act bigger and braver than she is.
QUESTION 480
When caring for a postoperative cholecystectomy client, the nurse assesses patency and
documents drainage of the T-tube. The nurse recognizes that the expected amount of drainage
during the first 24 hours postoperatively is:
A. 50–100 mL
B. 200–300 mL
C. 300–500 mL
D. 1000–1200 mL
Answer: C
Explanation:
(A) During the first 24 hours after surgery, the drainage is normally 300–500 mL and then
decreases to about 200 mL in 24 hours during the next 3–4 days.
(B) This range is the amount of drainage after the first 24 hours postoperatively. During the
first 24 hours, it is 300-500 mL.
(C) During the first 24 hours after surgery, this range is the expected amount of drainage.

(D) The expected amount of drainage during the first 24 hours is 300–500 mL. An output of
500 mL should be reported to the physician, because an occlusion of some type, caused by a
retained gallstone or an inflammatory process within the biliary drainage system, is evident.
QUESTION 481
A female client has been hospitalized for several months following major abdominal surgery
for a ruptured colon. A colostomy was created, and the large abdominal wound was left open
and allowed to heal through granulation. She is receiving gentamicin IV for treatment of
wound infection. Knowing this drug is ototoxic, the nurse would implement which of the
following measures?
A. Instruct the client to report any signs of tinnitus, dizziness or difficulty hearing.
B. Advise the client to discontinue the drug at the first sign of dizziness.
C. Order audiometric testing in order to determine if hearing loss is caused by an ototoxic
drug or other cause.
D. Instruct the client in Valsalva’s maneuver to equalize middle ear pressure and to prevent
hearing loss.
Answer: A
Explanation:
(A) The first nursing measure is to instruct the client in which drug side effects to report.
(B) Discontinuing the drug is not an independent nursing intervention and may compromise
client care.
(C) Audiometric testing will detect hearing loss, but it does not indicate a potential cause.
(D) Equalizing middle ear pressure will not prevent hearing loss.
QUESTION 482
The physician prescribes phenytoin (Dilantin) for a client with seizure disorders. Phenytoin
can only be mixed with which of the following solutions?
A. Ringer’s lactate
B. D5 in water
C. D5 with Ringer’s lactate
D. Normal saline
Answer: D
Explanation:
(A) Phenytoin will precipitate if mixed with Ringer’s lactate and should not be administered.

(B, C) Phenytoin will precipitate if mixed with D5 in Ringer’s lactate and should not be
administered.
(D) Phenytoin is compatible only with normal saline and should be mixed only with normal
saline for administration.
QUESTION 483
A postoperative prostatectomy client is preparing for discharge from the hospital the next
morning. The nurse realizes that additional instructions are necessary when he states:
A. “If I drink 10 to 12 glasses of fluids each day, that will help to prevent any clot formation
in my urine.”
B. “The isometric exercises will help to strengthen my perineal muscles and help me control
my urine.”
C. “If I feel as though I have developed a fever, I will take a rectal temperature, which is the
most accurate.”
D. “I do not plan to do any heavy lifting until I visit my doctor again.”
Answer: C
Explanation:
(A) This is correct health teaching. Drinking 10–12 glasses of clear liquid will help increase
urine volumes and prevent clot formation.
(B) This is correct health teaching. These types of exercises are prescribed by physicians to
assist postprostatectomy clients to strengthen their perineal muscles.
(C) This action is not recommended post-TURP because of the close proximity of the prostate
and rectum.
(D) This is correct healthcare teaching. The client should limit walking long distances, lifting
heavy objects, or driving a car until these activities are cleared by the physician at the first
office visit.
QUESTION 484
The nurse is caring for a client who has diabetes insipidus. The nurse would describe this
client’s urine output pattern as:
A. Anuria
B. Oliguria
C. Dysuria
D. Polyuria

Answer: D
Explanation:
(A) Anuria is defined as absence of urine output, which is not indicative of the urinary pattern
of diabetes insipidus.
(B) Oliguria is defined as 500 mL of urine per day, which is not a urinary output pattern
associated with diabetes insipidus.
(C) Dysuria is defined as difficult urination. Clients with diabetes insipidus do not have
dysuria as a symptom of their disease.
(D) Polyuria is a primary symptom of diabetes insipidus. These clients have decreased or
absent vasopressin secretion, which causes water loss in the urine and sodium increases.
QUESTION 485
The most frequent cause of early postpartum hemorrhage is:
A. Hematoma
B. Coagulation disorders
C. Uterine atony
D. Retained placental fragments
Answer: C
Explanation:
(A) Hematomas, which are the result of damage to a vessel wall without laceration of the
tissue, are a cause, though not the most frequent cause.
(B) Coagulation disorders are among the causes of postpartal hemorrhage, but they are less
common.
(C) The most frequent causes of hemorrhage in the postpartal period are related to an
interference with involution of the uterus. Uterine atony is the most frequent cause, occurring
in the first 24 hours after delivery.
(D) Retained placental fragments are also a cause, although these bleeds usually occur 7– 14
days after delivery.
QUESTION 486
A 42-year-old client presents with a diagnosis of paranoid schizophrenia. She has become
increasingly restless and verbally argumentative, and her speech has become pressured. She is
exhibiting signs of:
A. Depression

B. Agitation
C. Psychotic ideation
D. Anhedonia
Answer: B
Explanation:
(A) Signs of depression would include withdrawal, sadness, morbid thoughts, insomnia, early
awakening, etc.
(B) These clinical features are classic signs of agitation.
(C) Psychotic ideation includes delusional thoughts, bizarre behavior, disorganized thinking,
etc.
(D) Anhedonia is the inability to experience pleasure.
QUESTION 487
A client is having a vertical partial laryngectomy, and the nurse is planning his postoperative
care. A priority postoperative nursing diagnosis for a client having a vertical partial
laryngectomy would be:
A. Activity intolerance
B. Ineffective airway clearance
C. High risk for infection
D. Altered oral mucous membrane
Answer: B
Explanation:
(A) The laryngectomy client should be able to gradually increase activities without difficulty.
(B) The laryngectomy client may have copious amounts of secretions and require suctioning
for the first 24–48 hours. The cannula will require cleaning even after the first 24 hours
because mucus collects in it.
(C) The client does have a potential for infection, but it is not a more important nursing
priority than the ineffective airway clearance.
(D) This problem is not a more important nursing priority than ineffective airway clearance.
The client’s mouth may become dry, but good oral care should take care of the dryness.
QUESTION 488
A client who is a breast-feeding mother develops mastitis. The clinical signs and symptoms of
mastitis include:

A. Marked engorgement, elevated temperature, chills, and breast pain with an area that is red
and hardened
B. Marked engorgement and breast pain
C. Elevated temperature and general malaise
D. Cracked nipple with complaints of soreness
Answer: A
Explanation:
(A) Mastitis is a bacterial inflammation of the breast tissue found primarily in breast-feeding
mothers. The bacteria usually enter the breast through a cracked nipple, or the infection
results from stasis of milk behind a blocked duct.
(B) With breast engorgement during breast-feeding, there may be marked breast pain. This is
not necessarily a sign of infection.
(C) Women may become ill during breast-feeding with other bacterial or viral infections that
are not related to mastitis.
(D) Improper care of the nipples or improper positioning of the infant during breastfeeding
may result in cracked or sore nipples.
QUESTION 489
A client diagnosed with severe anemia is to receive 2 U of packed red blood cells. Prior to
starting the blood transfusion, the nurse must:
A. Take a baseline set of vital signs
B. Hang Ringer’s lactate as the companion fluid
C. Use micro drip tubing for the blood administration
D. Have the registered nurse in charge assume responsibility for verifying the client and
blood product information
Answer: A
Explanation:
(A) A baseline set of vital signs is necessary to determine if any transfusion reactions occur as
the blood product is being administered.
(B) The only companion fluid to be used during a blood transfusion is normal saline. The
calcium in Ringer’s lactate can cause clotting.
(C) Only a blood administration set should be used. A micro drip tube would cause lysis of
the red blood cells.

(D) Proper identification of the recipient and the blood product must be validated by at least
two people.
QUESTION 490
The nurse caring for a client who has pneumonia, which is caused by a gram-positive
bacteria, inspects her sputum. Because the client’s pneumonia is caused by a gram-positive
bacteria, the nurse experts to find the sputum to be:
A. Bright red with streaks
B. Rust colored
C. Green colored
D. Pink-tinged and frothy
Answer: B
Explanation:
(A) Bright red sputum with streaks is associated with pneumonia caused by gram-negative
bacteria, such as Klebsiella pneumonia.
(B) Pneumococcal pneumonia, caused by gram-positive bacteria, has a characteristic
productive cough with green or rust-colored sputum.
(C) Green-colored sputum is more characteristic of Pseudomonas than of gram-positive
bacterial pneumonia.
(D) Pink-tinged and frothy sputum is more characteristic of pulmonary edema than of grampositive bacterial pneumonia.
QUESTION 491
A client is experiencing mucosal cell damage secondary to chemotherapy. Because of
mucosal ulcers, eating has become increasingly uncomfortable for her. Which of the
following interventions would be most effective in getting her to eat?
A. Local anesthetics or mouth washes applied to ulcers 30 minutes prior to meals
B. A bland, moist, soft diet
C. Staying with the client and providing distraction during meals
D. Cleaning the mouth carefully with lemon glycerin swabs and milk of magnesia before
meals
Answer: B
Explanation:

(A) Local anesthetics do temporarily relieve the pain but leave an unpleasant taste and numb
feeling that are not conductive to eating.
(B) Such a diet is less irritating to the damaged mucosa and is easier for the child to tolerate.
(C) This intervention is helpful if the child has only anorexia. It does not work if the type and
texture of the food increase oral discomfort.
(D) Lemon glycerin swabs and milk of magnesia dry the oral mucosa and should be avoided.
QUESTION 492
A client had a hemicolectomy performed 2 days ago. Today, when the nurse assesses the
incision, a small part of the abdominal viscera is seen protruding through the incision. This
complication of wound healing is known as:
A. Excoriation
B. Dehiscence
C. Decortication
D. Evisceration
Answer: D
Explanation:
(A) Excoriation is abrasion of the epidermis or of the coating of any organ of the body by
trauma, chemicals, burns, or other causes.
(B) Dehiscence is a partial or complete separation of the wound edges with no protrusion of
abdominal tissue.
(C) Decortication is removal of the surface layer of an organ or structure. It is a type of
surgery, such as removing the fibrinous peel from the visceral pleura in thoracic surgery.
(D) Evisceration occurs when the incision separates and the contents of the cavity spill out.
QUESTION 493
A client has returned to the unit from the recovery room after having a thyroidectomy. The
nurse knows that a major complication after a thyroidectomy is:
A. Respiratory obstruction
B. Hypercalcemia
C. Fistula formation
D. Myxedema
Answer: A
Explanation:

(A) Respiratory obstruction due to edema of the glottis, bilateral laryngeal nerve damage, or
tracheal compression from hemorrhage is a major complication after a thyroidectomy.
(B) Hypocalcemia accompanied by tetany from accidental removal of one or more
parathyroid glands is a major complication, not hypercalcemia.
(C) Fistula formation is not a major complication associated with a thyroidectomy. It is a
major complication with a laryngectomy.
(D) Myxedema is hypothyroidism that occurs in adults and is not a complication of a
thyroidectomy. A thyroidectomy client tends to develop thyroid storm, which is excess
production of thyroid hormone.
QUESTION 494
A 48-year-old client is in the surgical intensive care unit after having had three-vessel
coronary artery bypass surgery yesterday. She is extubated, awake, alert and talking. She is
receiving digitalis for atrial arrhythmias. This morning serum electrolytes were drawn. Which
abnormality would require immediate intervention by the nurse after contacting the
physician?
A. Serum osmolality is elevated indicating hemoconcentration. The nurse should increase IV
fluid rate.
B. Serum sodium is low. The nurse should change IV fluids to normal saline.
C. Blood urea nitrogen is subnormal. The nurse should increase the protein in the client’s diet
as soon as possible.
D. Serum potassium is low. The nurse should administer KCl as ordered.
Answer: D
Explanation:
(A) An elevated serum osmolality poses no immediate danger and is not corrected rapidly.
(B) A low serum sodium alone does not warrant changing IV fluids to normal saline. Other
assessment parameters, such as hydration status, must be considered.
(C) A low serum blood urea nitrogen is not necessarily indicative of protein deprivation. It
may also be the result of overhydration.
(D) A low serum potassium potentiates the effects of digitalis, predisposing the client to
dangerous arrhythmias. It must be corrected immediately.

QUESTION 495
A client is going to have a pneumonectomy in the morning. She had a previous negative
surgical experience, is talking rapidly, and has an increased pulse and respiratory rate.
Nursing interventions for this client should include:
A. Providing opportunities to ask questions and talk about concerns
B. Providing distractors such as reading or watching television
C. Telling her that she should not be so nervous and assuring her that everything will be OK
D. Reminding her that this surgery is not as extensive as her past surgery was
Answer: A
Explanation:
(A) This intervention will help to clarify any misunderstandings about the surgery and give
the client an opportunity to verbalize concerns about the surgery.
(B) Distractors will not alleviate the preoperative anxiety that the client is experiencing. This
response actually denies the client’s anxiety.
(C) This intervention is false assurance and denies that anxiety is a normal response to the
threat of surgery.
(D) Psychological responses are not directly related to the extent of the surgery, because they
are influenced by the client’s past experiences.
QUESTION 496
Following her surgery, a 5-year-old child will return to the pediatric unit with a long-arm cast.
She experienced a supracondylar fracture of the humerus near the elbow. Which nursing
action is most essential during the first 24 hours after surgery and cast application?
A. Mobilization of the child
B. Discharge teaching
C. Pain management
D. Assessment of neurovascular status
Answer: D
Explanation:
(A) Mobilization is important but not absolutely essential.
(B) Discharge teaching should be initiated prior to surgery as well as during the postoperative
period.
(C) Assessment and management of pain are necessary and high in priority.

(D) Neurovascular status of the extremity is of primary importance. The risk of circulatory
impairment exists with any cast application. This type of fracture is common in children. A
high incidence of neurovascular complications exists with fractures near the elbow.
QUESTION 497
When giving discharge instructions to a 24-year-old client who had a short-arm cast applied
for a fractured right ulna, the nurse recognizes the importance of telling him that the drying
time for a plaster of Paris cast is approximately:
A. 30 minutes
B. 1–4 hours
C. 12–24 hours
D. 24–72 hours
Answer: D
Explanation:
(A) Synthetic cast materials harden in 3–15 minutes. Weight bearing is permitted in 15–30
minutes. Drying time for plaster of Paris is about 24–72 hours.
(B, C) Plaster of Paris cast materials are heavier than synthetic materials and require a drying
time of 24–72 hours. Synthetic materials dry within 30 minutes.
(D) Plaster of Paris cast materials are heavier than synthetic materials and require a longer
period to set and dry. Even though setting time (hardening) is only 3–15 minutes, the drying
time for plaster of Paris is 24–72 hours. This depends on the size and thickness of the cast,
exposure to air, and humidity in the air.
QUESTION 498
A female client has married recently. A month ago she visited her physician with complaints
of burning on urination. She was given a prescription for trimethoprim- sulfamethoxazole
(Bactrim) DS bid for 10 days. She was admitted through the emergency room on Saturday
evening complaining of flank pain. Her temperature was 104°F. A preliminary urinalysis
revealed 31 bacteria along with red and white blood cells in the urine. A preliminary
diagnosis of pyelonephritis was made. During a nursing admission assessment, which
statement by the client demonstrates a possible cause for pyelonephritis?
A. “I have not been drinking six to eight glasses of water each day as the nurse had
instructed.”

B. “I’m afraid I may have something wrong with my bladder because I have been getting
bladder infections frequently since I’ve been married.”
C. “I took the Bactrim for 6 or 7 days. The burning stopped, so I saved the rest of the
medication for the next time.”
D. “I recently had the flu, which could be settling in my kidneys now.”
Answer: C
Explanation:
(A) Although it is important that the client drink adequate fluids while treating a bladder
infection with trimethoprim sulfamethoxazole, the failure to do so will not cause
pyelonephritis.
(B) A stricture or abnormality may cause the progression of bladder infection to urinary tract
infection, but this is rare. There is no indication in this situation that this has occurred.
(C) The most common cause of pyelonephritis is improper treatment of bladder infections.
The client typically feels better after several days, discontinues the medication, and saves the
remainder for the next occurrence of a bladder infection. For this reason, it is imperative to
provide client education related to completion of the prescribed medication.
(D) There is no evidence that infection in another body system could cause pyelonephritis.
QUESTION 499
Endotracheal tube cuff pressure should never exceed:
A. 10 mm Hg
B. 20 mm Hg
C. 45 mm Hg
D. 60 mm Hg
Answer: B
Explanation:
(A) Pressure 10 mm Hg places the client at risk for aspiration.
(B) Pressure in the endotracheal tube cuff should never exceed 20 mm Hg, because higher
pressure places the client at risk for tracheal erosion.
(C) A pressure of 45 mm Hg is an extremely high pressure in the endotracheal tube cuff. This
places the client at great risk for tracheal erosion.
(D) A pressure of 60 mm Hg is an extremely high pressure in the endotracheal tube cuff. This
places the client at great risk for tracheal erosion.

QUESTION 500
Primary nursing diagnoses for the antisocial client are:
A. Alteration in perception and altered self-concept
B. Impaired social interaction, ineffective individual coping, and altered self-concept
C. Altered communication processes and altered recreational patterns
D. Altered body image and altered thought processes
Answer: B
Explanation:
(A) This answer is incorrect. Perception is not altered because the client is not psychotic.
(B) This answer is correct. The antisocial client lacks responsibility, accountability, and social
commitment; has impaired problem-solving ability; tends to overuse defense mechanisms;
lies and steals; and is often grandiose concerning self.
(C) This answer is incorrect. Altered communication processes do not characterize this client.
The antisocial person communicates well and tends to have a charming personality.
(D) This answer is incorrect. Altered thought processes refer to delusional thinking, which is
bizarre and fixed, and do not characterize this client.
QUESTION 501
A 71-year-old client fell and injured her left leg while cooking in the kitchen. Her husband
calls the ambulance, and she is taken to the emergency department at a local hospital. X-ray
reports confirm that she has an intertrochanteric fracture of the left femur. Her left leg will
require skeletal traction initially and then surgery. The nurse knows that this type of traction
will be used:
A. By inserting pins to provide steady pull on the bone
B. To suspend the leg in a sling without pull on the extremity
C. Intermittently to place a pull over the pelvis and lower spine
D. With weights at both ends of the bed to maintain pull on the upper extremity
Answer: A
Explanation:
(A) Skeletal traction is the application of traction directly to bone with the use of pins and
wires or tongs for the purpose of providing a strong, steady, continuous longitudinal pull on
the bone. It is indicated for preoperative immobilization and positioning of hip and femur
fractures.

(B) A type of skeletal traction (balanced suspension with a Thomas splint and Pearson
attachment) uses a sling to support the extremity, but it also uses weights to provide a strong,
steady continuous pull on the extremity. A sling is used instead of pins.
(C) Pelvic traction provides an intermittent pull over the pelvis and bone, whereas skeletal
traction is continuous. Pelvic traction does not use pins.
(D) Skeletal traction uses weights at the end of the bed to provide a continuous pull on long
bones. Weights are not applied to both ends of the bed.
QUESTION 502
In working with mental health clients who are prescribed medication that must be taken on a
routine basis, it is important for education to begin when the drug therapy is initiated. One of
the first steps in the teaching process is to:
A. Explain the side effects of the medication
B. Discuss the danger of overmedication
C. Distribute written material to supplement verbal instructions
D. Explore the client’s perception regarding medication therapy
Answer: D
Explanation:
(A, B, C) The nurse must first obtain information regarding the client’s perception of the
medication regimen.
(D) The first step in the teaching process is to determine the client’s perception.
QUESTION 503
A client has been uncomfortable in crowds all her life. After the birth of her child, she has
been housebound unless her husband can accompany her to the grocery store and for medical
appointments. His schedule will not allow for this, and he has insisted that she must be more
independent. Her anxiety has increased to the point of panic. The client has been diagnosed
with agoraphobia. Which statement is true about this disorder?
A. The behavior is not considered disabling.
B. More men suffer from agoraphobia than women.
C. The fears are persistent, and avoidance is used as the coping mechanism.
D. Agoraphobia moves into remission when treated with chlorpromazine.
Answer: C
Explanation:

(A) Agoraphobia is the most pervasive and serious phobic disorder.
(B) Women compose 70%–85% of agoraphobia sufferers.
(C) Agoraphobia is an acute disorder that immobilizes the sufferer with extreme anxiety.
(D) Chlorpromazine is not a drug used to treat phobias.
QUESTION 504
The nurse is planning a reality orientation program for a group of clients with organic brain
syndrome at the mental health center. Props that could be used for this program are:
A. Month-old magazines that are provided by volunteers
B. Large maps and posters depicting area of current residence
C. A litter of kittens for the clients to pet
D. A library of biographical books
Answer: B
Explanation:
(A) This answer is incorrect. Current magazines would be appropriate.
(B) This answer is correct. Maps of the state and town and posters that depict current events
in the area are appropriate props.
(C) This answer is incorrect. Kittens would be appropriate for pet therapy, not reality therapy.
(D) This answer is incorrect. Biographies depict a past, not a present, orientation.
QUESTION 505
An infant with a congenital heart defect is being discharged with an order for the
administration of digoxin elixir every 12 hours. The parents need to be taught when
administering digoxin to the infant that:
A. If the infant vomits within 30 minutes of the digoxin administration, repeat the dose
B. They need to mix it with formula so the infant swallows it easily
C. If the infant vomits two or more consecutive doses or becomes listless or anorexic, notify
the physician
D. If a dose of digoxin is skipped for more than 6 hours, a new timetable for administration
must be developed
Answer: C
Explanation:

(A) Occasionally the child may vomit. They should not repeat the dose because the amount of
digoxin that was absorbed is un-known, and serum levels of digoxin that are too high are
more dangerous than those that are temporarily too low.
(B) To ensure that the entire dose of digoxin is received, never mix it with food or formula.
(C) Vomiting, anorexia, and listlessness are all signs of digoxin toxicity and should be
reported to the physician immediately.
(D) If a dose is forgotten for more than 6 hours, the nurse should advise the parents to skip
that dose and to continue the next dose as scheduled.
QUESTION 506
A 3-year-old child was hospitalized for acute laryngotracheobronchitis. During her
hospitalization, the child was placed under an oxygen mist tent. The nurse’s frequent
monitoring of the child’s temperature frightened her parents. Which response by the nurse
would be most appropriate?
A. Monitoring the temperature prevents undue chilling.
B. Rapid temperature elevations can occur in children.
C. Checking the temperature will prevent febrile seizures.
D. Taking the child’s temperature can prevent airway obstruction.
Answer: A
Explanation:
(A) The refrigerated cool mist tent creates a cool, moist environment. The child as well as
bedding and clothing may become dampened. Monitoring the temperature of the child will
ensure warmth and prevent chilling.
(B) Only a low-grade fever is expected in laryngotracheobronchitis.
(C) Febrile seizures are not expected with the low-grade fever.
(D) Inflammation of the mucosal lining in the respiratory tract can cause airway obstruction.
However, monitoring the child’s temperature would not prevent airway obstruction.
QUESTION 507
A 72-year-old male client had the Foley catheter that was inserted during the transurethral
resection of his prostate removed today. He is concerned about the urinary incontinence he is
having since removal of the Foley catheter. The nurse explains that:
A. He should not be concerned about it because it will resolve quickly
B. This is usually temporary

C. The nurse will keep him dry, and he should notify the nurse when this happens
D. This is related to the bladder spasms and will soon stop
Answer: B
Explanation:
(A) This problem is temporary, but it may take some time to resolve, especially in an older
man.
(B) This problem is usually temporary, but it may take some time to resolve.
(C) Keeping the client dry will not relieve his anxiety about his incontinence.
(D) The bladder spasms are not the cause of the client’s incontinence.
QUESTION 508
A 48-year-old client presents with a long history of severe depression unrelieved by
medication. He is admitted to the hospital for electroconvulsive therapy. Family members are
very concerned about this therapy and are requesting information about aftereffects of the
treatment. The nurse informs the family that he will:
A. Have transient memory loss, confusion, and headache
B. Be alert and oriented immediately after the treatment
C. Have insomnia for the first few days
D. Require no special care after the procedure
Answer: A
Explanation:
(A) This answer is correct. The client will be confused and have a memory loss, which is
usually temporary, after electroconvulsive shock therapy.
(B) This answer is incorrect. The client will experience transient memory loss, look
bewildered, and be confused initially.
(C) This answer is incorrect. The client will sleep immediately following the treatment.
(D) This answer is incorrect. Vital signs are taken at least hourly after treatment. The client is
monitored for hypotension, tachycardia, respiratory problems, and possible seizure activity.
QUESTION 509
The nurse who is caring for a client with pneumonia assesses that the client has become
increasingly irritable and restless. The nurse realizes that this is a result of:
A. Prolonged bed rest
B. The client’s maintaining a semi-Fowler position

C. Cerebral hypoxia
D. IV fluids of 2.5–3 liters in 24 hours
Answer: C
Explanation:
(A) Maintaining bed rest helps to decrease the O2 needs of the tissues, which decreases
dyspnea and workload on the respiratory system.
(B) The semi-Fowler or high-Fowler position is necessary to aid in lessening pressure on the
diaphragm from the abdominal organs, which facilitates comfort and easier breathing
patterns.
(C) Cerebral hypoxia causes the client with pneumonia to be increasingly irritable and
restless and results from the client not obtaining enough O2 to meet metabolic needs.
(D) Proper hydration facilitates liquefaction of mucus trapped in the bronchioles and alveoli
and enhances expectoration. Unless contraindicated, a reasonable amount of IV fluids to be
administered is at least 2.5–3 liters in a 24-hour period.
QUESTION 510
A postoperative TURP client returns from the recovery room to the general surgery unit and is
in stable condition. One hour later the nurse assesses him and finds him to be confused and
disoriented. She recognizes that this is most likely caused by:
A. Hypovolemic shock
B. Hypokalemia
C. Hypernatremia
D. Hyponatremia
Answer: D
Explanation:
(A) Early signs of hypovolemic shock include hypotension, tachycardia, tachypnea, pallor,
and diaphoresis.
(B) Early signs of potassium depletion include muscular weakness or paralysis, tetany,
postural hypotension, weak pulse, shallow respirations, apathy, weak voice, and
electrocardiographic changes.
(C) Early signs of an elevated sodium level include dry oral mucous membranes, marked
thirst, hypertension, tachycardia, oliguria or anuria, anxiety, and agitation.
(D) This answer is correct. Important early clinical findings of a decreased sodium
concentration include confusion and disorientation. Hyponatremia can occur after a TURP

because absorption during surgery through the prostate veins can increase circulating blood
volume and decrease sodium concentration.
QUESTION 511
On an assessment of a client’s mouth, the nurse notices white patches on the buccal mucosa.
The nurse tries to obtain a sample for a culture, but the lesion cannot be rubbed off. The nurse
would suspect that this lesion is:
A. Xerosteromia
B. Candidiasis
C. Leukoplakia
D. Stomatitis
Answer: C
Explanation:
(A) Xerostomia is dry mouth.
(B) Candidiasis can be rubbed off, but it will bleed.
(C) Leukoplakia cannot be rubbed off.
(D) Stomatitis is caused by candidiasis and gram-negative bacteria.
QUESTION 512
A 28-year-old woman was admitted to the hospital for a thyroidectomy. Postoperatively she is
taken to the postanaesthetic care unit for several hours. In preparing for the client’s return to
her room, which nursing measure best demonstrates the nurse’s thorough understanding of
possible post thyroidectomy complications?
A. Dressings are placed at the bedside for dressing changes, which are to be done every 2
hours to best detect postoperative bleeding.
B. Narcotics are readily available and administered when the client returns to her room to
prevent excruciating pain.
C. A tracheostomy set, O2, and suction are available at the bedside.
D. The nurse should instruct the client as soon as possible on alternative means of
communication.
Answer: C
Explanation:

(A) Dressing changes are done as necessary for bleeding. However, frequently, postthyroidectomy bleeding may not be visible on the dressing, but blood may drain down the
back of the neck by gravity.
(B) Narcotics are administered for acute pain as necessary. They are not necessarily given on
return of the client to her room.
(C) The most serious post thyroidectomy complication is ineffective airway and breathing
pattern related to tracheal compression and edema. A tracheostomy set, O2, and suction
should be available at bedside for at least the first 24 hours postoperatively.
(D) Impaired verbal communication may occur due to laryngeal edema or nerve damage, but
most commonly, it occurs due to endotracheal intubation. The client is usually able to
communicate but is hoarse.
QUESTION 513
A nurse is taking a maternal history for a client at her first prenatal visit. Her pregnancy test
was positive, she has two living children, she had one spontaneous abortion, and one infant
died at the age of 3 months. Which of the following best describes the client at the present?
A. Gravida 4, para 2, ab 1
B. Gravida 5, para 3, ab 1
C. Gravida 5, para 4, ab 0
D. Gravida 4, para 3, ab 0
Answer: B
Explanation:
(A) This individual has been pregnant four times, delivered two children, and had one
abortion.
(B) Your client has been pregnant five times, delivered three children, and had one abortion.
(C) This individual has been pregnant five times, delivered four children, and has not had an
abortion.
(D) This individual has been pregnant four times, delivered three children, and has not had an
abortion.
QUESTION 514
A common complication of cirrhosis of the liver is prolonged bleeding. The nurse should be
prepared to administer?
A. Vitamin C

B. Vitamin K
C. Vitamin E
D. Vitamin A
Answer: B
Explanation:
(A) Vitamin C does not directly affect clotting.
(B) Vitamin K is a fat-soluble vitamin that depends on liver function for absorption. Vitamin
K is essential for clotting.
(C) Vitamin E does not directly affect clotting.
(D) Vitamin A does not directly affect clotting.
QUESTION 515
A 40-year-old client is admitted to the coronary care unit with chest pain and shortness of
breath. The physician diagnosed an anterior wall myocardial infarction. What tests should the
nurse anticipate?
A. Reticulocyte count, creatinine phosphokinase (CPK)
B. Aspartate transaminase, alanine transaminase
C. Sedimentation rate, WBC count
D. Lactic dehydrogenase, CPK
Answer: D
Explanation:
(A) Reticulocyte count measures the number of immature erythrocytes. CPK is an enzyme
released from injured myocardial tissue.
(B) Aspartate transaminase is an enzyme released from injured myocardial tissue. Alanine
transaminase is an enzyme released for general tissue destruction, which is specific for liver
injury.
(C) Sedimentation rate is a nonspecific test for inflammation.
(D) Lactic dehydrogenase and CPK are enzymes released from injured myocardial tissue.
QUESTION 516
A 32-year-old female client is being treated for Guillain- Barré syndrome. She complains of
gradually increasing muscle weakness over the past several days. She has noticed an
increased difficulty in ambulating and fell yesterday. When conducting a nursing assessment,
which finding would indicate a need for immediate further evaluation?

A. Complaints of a headache
B. Loss of superficial and deep tendon reflexes
C. Complaints of shortness of breath
D. Facial paralysis
Answer: C
Explanation:
(A) Headaches are not associated with Guillain-Barré syndrome.
(B) Loss of superficial and deep tendon reflexes is expected with this diagnosis.
(C) Complaints of shortness of breath must be further evaluated. Forty percent of all clients
have some detectable respiratory weakness and should be prepared for a possible
tracheostomy. Pneumonia is also a common complication of this syndrome.
(D) Facial paralysis is expected and is not considered abnormal.
QUESTION 517
Following the delivery of a healthy newborn, a client has developed thrombophlebitis and is
receiving heparin IV. What are the signs and symptoms of a heparin overdose for which the
nurse would need to observe during postpartum care of the client?
A. Dysuria
B. Epistaxis, hematuria, dysuria
C. Vertigo, hematuria, ecchymosis
D. Hematuria, ecchymosis, and epistaxis
Answer: D
Explanation:
(A) Dysuria is not a common symptom of heparin overdose.
(B) Although epistaxis and hematuria are common symptoms of heparin overdose, dysuria is
not.
(C) Vertigo is not a common symptom of heparin overdose.
(D) Hematuria, ecchymosis, and epistaxis are the most common signs and symptoms of a
heparin overdose. Others are thrombocytopenia, elevated liver enzymes, and local injection
site complications.
QUESTION 518
An obstructing stone in the renal pelvis or upper ureter causes:

A. Radiating pain into the urethra with labia pain experienced in females or testicular pain in
males
B. Urinary frequency and dysuria
C. Severe flank and abdominal pain with nausea, vomiting, diaphoresis, and pallor
D. Dull, aching, back pain
Answer: C
Explanation:
(A) Radiating pain in the urethra in both sexes, extending into the labia in females and into
the testicle or penis in the male, indicates a stone in the middle or lower segment of the ureter.
(B) Urinary frequency and dysuria are caused by a stone in the terminal segment of the ureter
within the bladder wall.
(C) An obstructing stone in the renal pelvis or upper ureter causes severe flank and abdominal
pain with nausea, vomiting, diaphoresis, and pallor.
(D) Dull and aching pain may indicate early stages of hydronephrosis. Also, a stone in the
renal pelvis or upper ureter causes severe flank and abdominal pain.
QUESTION 519
A 15-year-old client is admitted to the adolescent unit. The nurse recognizes that encouraging
a client to speak openly depends on how clearly questions are phrased. Which of the
following statements is most desirable in eliciting information from an adolescent client?
A. “Do you get along well with your family?”
B. “Do you communicate with your parents?”
C. “You don’t hate your family, do you?”
D. “What is it like between you and your family?”
Answer: D
Explanation:
(A, B) This statement can be answered with a simple yes or no.
(C) This statement is asked in a negative manner and therefore has a negative connotation.
(D) This statement is open ended and positively stated.
QUESTION 520
A client is diagnosed with diabetic ketoacidosis. The nurse should be prepared to administer
which of the following IV solutions?
A. D5in normal saline

B. D5W
C. 0.9 normal saline
D. D5in lactated Ringer’s
Answer: C
Explanation:
(A) D5in normal saline would increase serum glucose.
(B) D5W would increase serum glucose.
(C) A concentration of 0.9 NS is used to correct extracellular fluid depletion.
(D) D5in Ringer’s lactate would increase serum glucose.
QUESTION 521
As a nurse works with an adolescent with cystic fibrosis, the nurse begins to notice that he
appears depressed and talks about suicide and feelings of worthlessness. This is an important
factor to consider in planning for his care because:
A. It may be a bid for attention and an indication that more diversionary activity should be
planned for him
B. No threat of suicide should be ignored or challenged in any way
C. He needs to be observed carefully for signs that his depression has been relieved
D. He needs to be confronted with his feelings and forced to work through them
Answer: B
Explanation:
(A) Threats of suicide should always be taken seriously.
(B) This client has a life-threatening chronic illness. He may be concerned about dying or he
may actually be contemplating suicide.
(C) Sometimes clients who have made the decision to commit suicide appear to be less
depressed.
(D) Forcing him to look at his feelings may cause him to build a defense against the
depression with behavioral or psychosomatic disturbances.
QUESTION 522
A 49-year-old obese woman has been admitted to the general surgery unit with
choledocholithiasis. As the nurse is admitting her to the unit, she states, “The doctor said I
have stones that need to be removed; where are they?” The nurse knows that the best
explanation for this is to tell her that:

A. There are stones present in her gallbladder
B. There are stones present in her kidneys
C. There are stones present in her common bile duct
D. There are no stones, but her gallbladder is irritated and caused her nausea, vomiting, and
pain
Answer: C
Explanation:
(A) Cholelithiasis is the correct term used to describe the presence of stones in the
gallbladder.
(B) Nephrolithiasis, or renal calculi, is the correct term used to describe the presence of
stones in the kidney.
(C) Choledocholithiasis as is the correct term used to describe the presence of stones in the
common bile duct.
(D) Cholecystitis is the correct term used to describe inflammation of the gallbladder and can
be associated with cystic duct obstructions from impacted stones.
QUESTION 523
A postoperative TURP client is ordered continuous bladder irrigations. Later in the evening
on the first postoperative day, he complains of increasing suprapubic pain. When assessing
the client, the nurse notes diminished flow of bloody urine and several large blood clots in the
drainage tubing. Which one of the following should be the initial nursing intervention?
A. Call the physician about the problem.
B. Irrigate the Foley catheter.
C. Change the Foley catheter.
D. Administer a prescribed narcotic analgesic.
Answer: B
Explanation:
(A) The physician should be notified as problems arise, but in this case, the nurse can attempt
to irrigate the Foley catheter first and call the physician if irrigation is unsuccessful. Notifying
the physician of problems is a subsequent nursing intervention.
(B) This answer is correct. Assessing catheter patency and irrigating as prescribed are the
initial priorities to maintain continuous bladder irrigation. Manual irrigation will dislodge
blood clots that have blocked the catheter and prevent problems of bladder distention, pain,
and possibly fresh bleeding.

(C) The Foley catheter would not be changed as an initial nursing intervention, but irrigation
of the catheter should be done as ordered to dislodge clots that interfere with patency.
(D) Even though the client complains of increasing suprapubic pain, administration of a
prescribed narcotic analgesic is not the initial priority. The effect of the medication may mask
the symptoms of a distended bladder and lead to more serious complications.
QUESTION 524
The health team needs to realize that the compulsive concern with cleanliness that a client
with severe anxiety exhibits is most likely an attempt to:
A. Reduce his anxiety
B. Avoid going to psychotherapy
C. Manipulate the health team members
D. Increase his self-image by showing higher standards than the fellow clients
Answer: A
Explanation:
(A) These behaviors are attempts to relieve anxiety.
(B) Avoidance is not a pattern in the obsessive client.
(C) Although these behaviors may seem to manipulate others, that is not the purpose behind
the activity.
(D) Inflated self-esteem is not a characteristic of the severely anxious client.
QUESTION 525
A 74-year-old client seen in the emergency room is exhibiting signs of delirium. His family
states that he has not slept, eaten, or taken fluids for the past 24 hours. The planning of
nursing care for a delirious client is based on which of the following premises?
A. The delirious client is capable of returning to his previous level of functioning.
B. The delirious client is incapable of returning to his previous level of functioning.
C. Delirium entails progressive intellectual and behavioral deterioration.
D. Delirium is an insidious process.
Answer: A
Explanation:
(A) This answer is correct. If the cause is removed, the delirious client will recover
completely.

(B) This answer is incorrect. The demented client is incapable of returning to previous level
of functioning. The delirious client is capable of returning to previous functioning.
(C) This answer is incorrect. The demented client, not the delirious client, has progressive
intellectual and behavioral deterioration.
(D) This answer is incorrect. Delirium develops rapidly, whereas dementia is insidious.
QUESTION 526
A client is experiencing visual problems at school. She has complained of difficulty seeing
the blackboard and squinting. She no longer likes to participate in physical activities such as
softball. The client has displayed possible classic symptoms of which refractive error?
A. Astigmatism
B. Hyperopia
C. Myopia
D. Amblyopia
Answer: C
Explanation:
(A) Visual images are blurred and distorted.
(B) Symptoms are headaches, burning eyes, fatigue, squinting, and difficulty reading.
(C) These symptoms are classic for myopia.
(D) Amblyopia is not a refractive error. It is a loss of vision in one or both eyes.
QUESTION 527
A 22-year-old client is 16 weeks pregnant. She and her husband are expecting their first baby.
The client tells the nurse that her last normal menstrual period was February 16, with 3 days
of spotting on February 17, 18, and 19. The nurse calculates her expected date of delivery to
be:
A. November 23rd
B. December 26th
C. September 14th
D. December 9th
Answer: A
Explanation:
(A) Naele’s rule is as follows: add 7 days to the 1st day of the last menstrual period, subtract
3 months, and then add 1 year.

(B) Naele’s rule presumes that the woman has a 28-day menstrual cycle, with conception
occurring on the 14th day of the cycle. Slight vaginal spotting may occur in early gestation
for unknown reasons but is insignificant in the calculation of Naele’s rule.
(C) Naele’s rule presumes that the woman has a 28-day menstrual cycle, with conception
occurring on the 14th day of the cycle. Slight vaginal spotting may occur in early gestation
for unknown reasons but is insignificant in the calculation of Naele’s rule.
(D) Naele’s rule presumes that the woman has a 28-day menstrual cycle, with conception
occurring on the 14thday of the cycle. Slight vaginal spotting may occur in early gestation for
unknown reasons but is insignificant in the calculation of Naele’s rule.
QUESTION 528
A female client was employed as a client care technician in a hemodialysis unit. She recently
began to experience extreme fatigue, being able to sleep for 16–20 hours at a time. She also
noted that her urine was tea colored, which she rationalized was a result of the vitamins she
began taking to alleviate fatigue. She was diagnosed with hepatitis B. After a brief hospital
stay, she is discharged to her parent’s home. Her mother asks the nurse if any precautions are
necessary to prevent transmission to the client’s family. The nurse explains necessary
precautions, which include:
A. Isolation of the client from the remainder of the family
B. Separate bathroom facilities if possible; if not, then cleansing daily of the facilities with a
chloride solution
C. No necessary precautions because she is beyond the contagious phase
D. Laundering clothes separately in cold water with a chloride solution
Answer: B
Explanation:
(A) Isolation is not necessary, even in the acute phase.
(B) Separate bathroom facilities are recommended. If unavailable, daily cleansing with a
chloride solution is recommended.
(C) Precautions continue to be necessary while the client is in the active phase of hepatitis.
(D) Clothes are to be laundered separately in hot water with a chloride solution.
QUESTION 529
A 30-year-old female client is receiving antineoplastic chemotherapy. Which of the following
symptoms should especially concern the nurse when caring for her?

A. Respiratory rate of 16 breaths/min
B. Pulse rate of 80 bpm
C. Complaints of muscle aches
D. A sore throat
Answer: D
Explanation:
(A) A respiratory rate of 16 breaths/min is normal and is not a cause for alarm.
(B) A pulse rate of 80 bpm is normal and is not a cause for alarm.
(C) Complaints of muscle aches are unrelated to her receiving chemotherapy. There may be
other causes related to her hospital stay or the disease process.
(D) A sore throat is an indication of a possible infection. A client receiving chemotherapy is at
risk of neutropenia. An infection in the presence of neutropenia can result in a life-threatening
situation.
QUESTION 530
A female client presents to the obstetric-gynecology clinic for a pregnancy test, the result
which turns out to be positive. Her last menstrual period began December 10, 1993. Using
Nägele’s rule, the nurse estimates her date of delivery to be:
A. September 17, 1994
B. September 10, 1994
C. September 3, 1994
D. August 17, 1994
Answer: A
Explanation:
(A) According to Nägele’s rule, the estimated date of delivery is calculated by adding 7 days
to the date of the first day of the normal menstrual period (December 10 + 7 days = December
17), and then by counting back 3 months (December 17 -3 mo = September 17).
(B, C, D) These answers are incorrect.
QUESTION 531
A female client has a chest tube placed. It is accidentally pulled out of the intrapleural space
when she is ambulating. The first action the nurse should take is to:
A. Instruct the client to cough deeply to re-expand her lung
B. Put on sterile gloves and replace the tube

C. Apply a petrolatum dressing over the site
D. Auscultate the lung to determine if she needs the tube replaced
Answer: C
Explanation:
(A) This action is inappropriate. Coughing will not re-expand the lung and could result in
further harm.
(B) This action is a medical procedure, not a nursing procedure.
(C) An occlusive dressing will prevent further air leak until the physician institutes further
treatment.
(D) The decision to reinsert the tube is a medical decision, not a nursing one.
QUESTION 532
The nurse observes a client crying quietly. She has just experienced a spontaneous abortion at
nine week’s gestation. An appropriate response by the nurse would be:
A. “It must be God’s will and probably is for the best.”
B. “This must be a difficult time for you. Would you like to talk about it?”
C. “I’m sure your other children will be a comfort for you.”
D. “Don’t worry, you’re still young. If I were you I’d just try again.”
Answer: B
Explanation:
(A) This response is nontherapeutic because it belittles the client’s response and gives a
meaningless rationalization.
(B) This response acknowledges the client’s feelings and demonstrates the therapeutic
offering of self by the nurse.
(C) This response is nontherapeutic because it does not focus on the client’s feelings and
offers false reassurance.
(D) This response is nontherapeutic because it belittles the client’s feelings and offers her
advice.
QUESTION 533
A client is dilated 8 cm and entering the transition phase of labor. Common behaviors of the
laboring woman during transition are:
A. Frustration, vague in communication
B. Seriousness, some difficulty following directions

C. Calmness, follows directions easily
D. Excitement, openness to instructions
Answer: A
Explanation:
(A) During the transition phase, the mother may become frustrated and unclear in her
communication owing to severe pain and fear of loss of control.
(B) These behaviors are common in the active phase of labor.
(C) These behavioral clues are seen in the latent phase of labor.
(D) These characteristics are observed in the latent phase of labor.
QUESTION 534
A 5-year-old has just had a tonsillectomy and adenoidectomy. Which of these nursing
measures should be included in the postoperative care?
A. Encourage the child to cough up blood if present.
B. Give warm clear liquids when fully alert.
C. Have child gargle and do toothbrushing to remove old blood.
D. Observe for evidence of bleeding.
Answer: D
Explanation:
(A) The nurse should discourage the child from coughing, clearing the throat, or putting
objects in his mouth. These may induce bleeding.
(B) Cool, clear liquids may be given when child is fully alert. Warm liquids may dislodge a
blood clot. The nurse should avoid red- or brown-colored liquids to distinguish fresh or old
blood from ingested liquid should the child vomit.
(C) Gargles and vigorous toothbrushing could initiate bleeding.
(D) Postoperative hemorrhage, though unusual, may occur. The nurse should observe for
bleeding by looking directly into the throat and for vomiting of bright red blood, continuous
swallowing, and changes in vital signs.
QUESTION 535
A primipara is assessed on arrival to the postpartum unit. The nurse finds her uterus to be
boggy. The nurse’s first action should be to:
A. Call the physician
B. Assess her vital signs

C. Give the prescribed oxytocic drug
D. Massage her fundus
Answer: D
Explanation:
(A) The nurse should first implement independent and dependent measures to achieve uterine
tone before calling the physician.
(B) Assessment of vital signs will not help to restore uterine atony, which is the priority need.
(C) Giving a prescribed oxytocic drug would be necessary if the uterus did not maintain tone
with massage.
(D) Fundal massage generally restores uterine tone within a few moments and should be
attempted first.
QUESTION 536
A 19-year-old primigravida is admitted to the labor and delivery suite of the hospital. Her
husband is accompanying her. The couple tells the nurse that this is the first hospital
admission for her. The client’s vaginal exam indicates she is 3 cm dilated, 80% effaced, and at
0 station. Based on the vaginal exam, she is in:
A. Stage 2, latent phase
B. Stage 1, active phase
C. Stage 3, transition phase
D. Stage 1, latent phase
Answer: D
Explanation:
(A) The second stage of labor is from full cervical dilation through birth of the baby. The
three phases of this stage include latency or resting, descent, and final transition. The client is
less than fully dilated so she is not in stage 2.
(B) The first stage of labor begins with regular uterine contractions and continues until the
woman is 10 cm dilated. The three phases of this stage include the early or latent phase (0–3
cm), the active phase (4–7 cm), and the transitional phase (7–10 cm). The client is 4 cm
dilated so she is in the latent phase of the first stage of labor.
(C) The third stage of labor is from the birth of the baby until the delivery of the placenta.
The client is less than fully dilated.
(D) The first stage of labor begins with regular uterine contractions and continues until the
woman is 10 cm dilated. The three phases of this stage include the early or latent phase (0–3

cm), the active phase (4–7 cm), and the transitional phase (7–10 cm). The client is 4 cm
dilated so she is in the latent phase of the first stage of labor.
QUESTION 537
A 10-year-old boy has been diagnosed with Legg-Calvé Perthes disease. Which of the client’s
responses would indicate compliance during initial therapy?
A. Drinking large amounts of milk
B. Not bearing weight on affected extremity
C. Walking short distances 3 times/day
D. Putting self on weight reduction diet
Answer: B
Explanation:
(A) This condition causes aseptic necrosis of the head of the femur in the acetabulum.
Drinking large quantities of milk at this time cannot hasten recovery.
(B) The aim of treatment is to keep the head of the femur in the acetabulum. Non–weightbearing is essential. Activity causes microfractures of the epiphysis.
(C) In addition to non–weight-bearing, clients are often placed on bedrest, which helps to
reduce inflammation. Later, active motion is encouraged.
(D) Weight is not generally an issue with this disease. Slipped femoral capital epiphysis,
which is most frequently observed in obese pubescent children, usually requires a weight
reduction diet.
QUESTION 538
A 32-year-old male client is a marketing representative. His job requires him to have a
tremendous amount of energy during the day. He frequently uses cocaine to sustain his energy
level. Lately he has increased his use of cocaine and even experimented with crack cocaine.
Realizing he can no longer continue this destructive behavior, he is seeking treatment for
cocaine addiction. In planning nursing care for the client’s inpatient stay, which expected
outcome is most appropriate?
A. He will attend four consecutive group educational sessions on substance abuse.
B. He will name activities that he would most likely be involved in posttreatment.
C. He will meet with his family in counseling sessions and discuss his feelings.
D. He will be able to deal with his feelings through participation in group therapy sessions.
Answer: D

Explanation:
(A) This expected outcome is specific as related to attendance, but not specific as related to
outcome criteria.
(B) Stating activities does not guarantee involvement.
(C) This goal may help the recovery process, but post counseling behavior is not addressed.
(D) This statement best describes the expected outcome. The client will be attending group
therapy sessions and through them he will deal with his feelings.
QUESTION 539
A 12-year-old girl has been diagnosed with insulin dependent diabetes mellitus. Which of
these principles would best guide her nutritional management?
A. Concentrated sweets are taken during increased activity.
B. Food restriction is imposed to reduce weight.
C. Caloric distribution should be calculated to fit activity patterns.
D. Fat requirements are increased owing to the possibility of ketoacidosis.
Answer: C
Explanation:
(A) Concentrated sweets are eliminated from diet planning. Complex carbohydrates may be
taken at the time of increased activity.
(B) Food restriction is not used for diabetic control of growing children. Caloric restriction
may be imposed for weight control if necessary.
(C) Total caloric intake and proportions of basic nutrients should be consistent from day to
day. Distribution of these calories should fit the activity pattern. Extra food is needed for
increased activity. A balance of food, exercise, and insulin should be maintained.
(D) Because of the increased risk of atherosclerosis, the fat percentage of the total caloric
intake is reduced.
QUESTION 540
A female client at 36 week’s gestation is experiencing preterm labor. Her physician has
prescribed two doses of betamethasone 12 mg IM q24h. The nurse explains that she is
receiving this drug to:
A. Treat fetal respiratory distress syndrome
B. Prevent uterine infection
C. Promote fetal lung maturation

D. Increase uteroplacental circulation
Answer: C
Explanation:
(A) Respiratory distress syndrome occurs in the newborn, not the fetus. It may be treated
postnatally with surfactant therapy.
(B) Betamethasone is a corticosteroid, not an anti-infective drug; therefore, its use would not
prevent uterine infection.
(C) Betamethasone binds with glucocorticoid receptors in alveolar cells to increase
production of surfactant, thus increasing lung maturity in the preterm fetus.
(D) Betamethasone does not affect uteroplacental circulatory exchange.
QUESTION 541
A client is pregnant for the fourth time and has had three normal vaginal deliveries. She is in
active labor and fully dilated. Suddenly she calls, “Nurse, the baby is coming”. As the nurse
responds to her call, which one of the following observations should the nurse make first?
A. Inspect the perineum.
B. Time the contractions.
C. Prepare a sterile area for delivery.
D. Auscultate for fetal heart rate (FHR).
Answer: A
Explanation:
(A) The nurse must assess the labor status to determine if birth is imminent. The nurse may
note perineal bulging, crowning, or birth of the head to ascertain labor status.
(B) Assessing uterine contractions is one intervention to ascertain labor status. Based on the
client’s cry, it is not the intervention of choice.
(C) If delivery of the infant is imminent, preparing a clean or sterile area for delivery is
appropriate, but labor status must be established, whether delivery is imminent, by perineal
assessment.
(D) Assessing FHR is one intervention to ascertain fetal well-being. Based on the client’s cry,
this is not the intervention of choice.
QUESTION 542
When interviewing parents who are suspected of child abuse, the nurse would use which of
the following interview techniques?

A. Be direct, honest, and attentive.
B. Approach them in the emergency room as soon as you suspect abuse to “clear the air right
away.”
C. Ask the parents what they could have done differently to prevent this from happening to
the child.
D. After the interview, call child protective services.
Answer: A
Explanation:
(A) The nurse must be honest, direct, professional, and attentive in her interview to gain the
parent’s trust.
(B) The nurse should approach the parents in private, away from the child.
(C) Asking them to relive and evaluate the situation may be looked at as placing blame on the
parents for the child’s “accident”. At this point, the parents may get defensive and stop
communicating.
(D) Although you may call child protective services, the nurse should inform the parents of
their responsibility to do this and explain the process to them.
QUESTION 543
An 82-year-old former restaurant owner walks to the nursing station and states, “I have to go.
The restaurant opens at 11 am”. Which response by the nurse is the most appropriate?
A. “Go back to your room. You do not own a restaurant.”
B. “You are in the hospital now. Calm down.”
C. “You once owned a restaurant. Tell me about it.”
D. “It is snowing outside. The restaurant is closed.”
Answer: C
Explanation:
(A) This response cuts off communication with the client. It does not address her feelings.
(B) Reality orientation frequently does not work alone. Feelings must be addressed. Telling a
client to calm down is frequently ineffective.
(C) Reminiscence is used here to reorient and recall past pleasant events. Talking about the
restaurant will allay anxiety.
(D) This response may confirm to the client that she indeed does still own a restaurant,
buying into her confusion. Her feelings and anxiety require nursing intervention.

QUESTION 544
A female client has been diagnosed with chronic renal failure. She is a candidate for either
peritoneal dialysis or hemodialysis and must make a choice between the two. Which
information should the nurse give her to help her decide?
A. Hemodialysis involves less time to filter the blood; but the client must consider travel
time, distance, and inconvenience.
B. Hemodialysis involves more time to filter the blood than does peritoneal dialysis.
C. Peritoneal dialysis has almost no complications and is less time consuming than
hemodialysis. Therefore it is preferred.
D. Peritoneal dialysis requires that a home health nurse prepare and administer the treatments.
Answer: A
Explanation:
(A) Hemodialysis is faster in clearing the blood of toxins than peritoneal dialysis. However,
clients must consider the time that they spend traveling to the dialysis center and the
disruption in their daily lives.
(B) Peritoneal dialysis requires several exchanges with dwelling time for the dialysate and
therefore takes longer than hemodialysis.
(C) Several serious complications of peritoneal dialysis include peritonitis, catheter
displacement and/or plugging, or pain during dialysis.
(D) A client can be taught to self-administer peritoneal dialysis without the aid of a
professional.
QUESTION 545
A 24-year-old graduate student recognizes that he has a phobia. He suffers severe anxiety
when he is in darkness. It has altered his lifestyle because he is unable to go to a movie
theater, concert, and other events that may require absence of light. The client is seeking
assistance because he is no longer able to socialize with friends due to his phobia. The
psychologist working with him is using desensitization. He has asked the nursing staff to
assist the client in muscle relaxation techniques. What result would indicate client education
has been successful?
A. He enters a movie theater, sits in his chair, and replaces anxiety with relaxation as the
theater darkens.
B. He enters a concert, but as the lights dim, he does not experience anxiety.

C. He states that he no longer fears dark places.
D. He takes a part-time job as a photographic assistant. His job necessitates his working in a
darkroom.
Answer: A
Explanation:
(A) This situation provides specific evidence that the client is able to integrate muscle
relaxation technique into his lifestyle to alleviate anxiety.
(B) The client may not experience anxiety at the concert, but there is no evidence regarding
the technique that he used to alleviate anxiety.
(C) The client may state he no longer experiences anxiety, but there is no evidence
demonstrating this. He may be denying anxiety to discontinue therapy prematurely.
(D) Does he experience anxiety in the darkroom? He may have taken this job to force himself
to deal with the phobia directly.
QUESTION 546
A female client is exhibiting signs of respiratory distress. Which of the following signs
indicate a possible pneumothorax?
A. Crackles or rales on the affected side
B. Bradypnea and bradycardia
C. Shortness of breath and sharp pain on the affected side
D. Increased breath sounds on the affected side
Answer: C
Explanation:
(A) With a pneumothorax, air occupies the pleural space. Crackles or rales are heard with
increased fluid or secretions and would not be present with air in the space.
(B) With a pneumothorax, the client would experience tachypnea and tachycardia to
compensate for the decrease in oxygenation.
(C) Symptoms of pneumothorax include shortness of breath, sharp pain on the affected side
with movement or coughing, asymmetrical chest expansion, and diminished or absent breath
sounds on the affected side.
(D) With a pneumothorax, breath sounds would be decreased on the affected side (indicates
air in the pleural space).

QUESTION 547
After instructing a female client on circumcision care, the nursery nurse asks her to restate
some of the key points covered. Which statement shows that the client will properly care for
her son’s circumcision?
A. “I’ll make sure I soak the gauze with warm water first, before I take it off each time.”
B. “I’ll make sure that I report any drainage around where they operated.”
C. “I’ll apply alcohol to the area daily to clean it and prevent any infection.”
D. “I’ll keep a close watch on it for a day or two.”
Answer: A
Explanation:
(A) Before petrolatum gauze is removed, it should be soaked with warm water to prevent
trauma to adherent tissues.
(B) A yellow exudate often forms normally over the surgical site. Only if it becomes foulsmelling and purulent would it need to be reported.
(C) Alcohol should never be used on the site; this would be extremely painful to the infant.
(D) Special care and observance should continue until the site is completely covered with
clean, pink granulation tissue, which could take 7–10 days.
QUESTION 548
An 11-month-old infant is admitted with a possible diagnosis of pyloric stenosis. Which of
the following best describes the characteristic clinical manifestations of pyloric stenosis?
A. Pain, especially when eating
B. Poor appetite and sucking reflex
C. Increased frequency and quantity of stools
D. Palpable olive-shaped mass in the epigastrium just right of the umbilical cord
Answer: D
Explanation:
(A) There is no evidence of pain in infants with pyloric stenosis whether eating or not.
(B) There are both good appetite and feeding habits in these children.
(C) Because of regurgitation, there is usually decreased frequency and quantity of stools and
also signs of dehydration and weight loss.
(D) Along with upper abdominal distention, there is a characteristic palpable olive-shaped
mass located to the right of the umbilicus.

QUESTION 549
A 15-year-old female adolescent is frequently breaking the rules of the unit. She has left the
unit and was found smoking in the bathroom and spending a large amount of time in the male
ward. Which statement by the nurse would best explain to the teenager why she must follow
the rules of the unit?
A. “It is not easy, but the rules must be followed so that everyone can get a fair chance.”
B. “If you do not follow the rules, you will be transferred to the closed, locked unit.”
C. “You are not being fair to the other clients by getting them involved in your deviant
behavior.”
D. “Break the rules, all you want, but don’t get caught again!”
Answer: A
Explanation:
(A) This statement acknowledges that it is difficult but is not threatening or punitive.
(B) This statement is threatening and describes specific punishment for further deviant
behavior.
(C) This response elicits shame by blaming her for involving others.
(D) This response gives her permission to break the rules but indicates that getting caught is
wrong.
QUESTION 550
A male client is admitted to the medical-surgical unit from the emergency room with a
diagnosis of acute pancreatitis. The nurse performs the admission nursing assessment. He is
NPO with IV fluids infusing at 100 mL/hour. He is experiencing excruciating abdominal pain.
Based on an analysis of these data, which nursing diagnosis would receive the highest
priority?
A. Pain related to stimulation of nerve endings associated with obstruction of the pancreatic
tract
B. Fluid volume deficit related to vomiting and nasogastric tube drainage
C. Knowledge deficit related to treatment regimen
D. Altered nutrition: less than body requirements, related to inadequate intake associated with
current anorexia, nausea, vomiting, and digestive enzyme loss
Answer: A
Explanation:

(A) Relief of pain is the primary goal of nursing intervention because this client is
experiencing acute pain.
(B) Fluid volume deficit is being treated with IV fluid replacement.
(C) Knowledge deficit will not be addressed at this time because a client in acute pain is not
ready to learn.
(D) Alteration in nutrition is the third priority after relief of pain and fluid volume deficit.
QUESTION 551
A 20-year-old client presents to the obstetrics-gynecology clinic for the first time. She tells
the nurse that she is pregnant and wants to start prenatal care. After collecting some initial
assessment data, the nurse measures her fundal height to be at the level of the umbilicus. The
nurse estimates the fetal gestational age to be approximately:
A. 10 weeks
B. 16 weeks
C. 20 weeks
D. 30 weeks
Answer: C
Explanation:
(A) At 10 weeks, the fundus is located slightly above the symphysis pubis.
(B) At 16 weeks, the fundus is halfway between the symphysis pubis and the umbilicus.
(C) At 20 weeks, the fundus is located approximately at the umbilicus.
(D) At 30 weeks, the fundal height is about 30 cm, or 10 cm above the umbilicus.
QUESTION 552
A male client has been an insulin-dependent diabetic for approximately 30 years. He
frequently indulges in high sugar foods and forgets to take his insulin. He has not experienced
acute diabetic emergencies over the years but is now beginning to demonstrate symptoms of
diabetic peripheral neuropathy. This distresses him because dancing is one of his favorite
pastimes. He decides to question his wife’s home health nurse about diabetic peripheral
neuropathy. The nurse points out his noncompliance to his diabetic diet and insulin regimen.
The client answers the nurse, “It has been my experience that the diabetic diet is very difficult
to follow.” As far as the insulin, isn’t a fellow allowed to forget now and then? The client’s
actions and response best demonstrate:
A. Depression

B. Anger
C. Denial
D. Bargaining
Answer: C
Explanation:
(A) Depression may be an underlying feature, but it is not evident from limited data presented
here.
(B) Anger is not exhibited in his response.
(C) Denial is evident in the client’s actions; through the years, he has had a casual approach to
his illness. He only becomes concerned when bodily changes affect his present lifestyle, when
in fact he should have been concerned all along. His verbal response also reflects denial.
(D) There is no evidence of bargaining in the client’s actions or verbal response.
QUESTION 553
A 10-year-old has been diagnosed with acute poststreptococcal glomerulonephritis. The
clinical findings were proteinuria, moderately elevated blood pressure, and periorbital edema.
Which dietary plan is most appropriate for this client?
A. Low-protein diet
B. Low-sodium diet
C. Increased fluid intake
D. High-cholesterol diet
Answer: B
Explanation:
(A) A high-protein diet is usually indicated because protein is excreted in urine. Protein
restriction is usually prescribed with severe azotemia.
(B) The kidneys usually enlarge in these children, and sodium and water are retained.
(C) Fluid restriction may be ordered to help reduce edema; however, monitoring for
dehydration is indicated.
(D) A high-cholesterol diet would not be indicated for any child, especially one with elevated
blood pressure.

QUESTION 554
A 50-year-old male client is to receive chemotherapy. The physician’s orders include
antiemetics. When planning his care, the nurse should take into consideration that antiemetics
are best administered in the following way:
A. Give antiemetics when nausea is experienced and continue on a regular schedule for 12–
24 hours.
B. Give antiemetics prior to the client receiving chemotherapy and continue on a regular basis
for at least 24–48 hours after chemotherapy.
C. Give antiemetics one at a time because combinations of antiemetics cause overwhelming
side effects.
D. Give antiemetics intermittently during the entire course of chemotherapy.
Answer: B
Explanation:
(A) Nausea is more difficult to control if antiemetics are withheld until nausea is experienced.
(B) Antiemetics should be given prophylactically at the beginning of chemotherapy and
continued on an around-the-clock basis to prevent nausea.
(C) Combinations of antiemetics give the best control for nausea by blocking various causes
of nausea induced by chemotherapy.
(D) Antiemetics should be given around the clock during the course of chemotherapy. This
prevents nausea from developing and prevents anticipatory nausea during subsequent
chemotherapy administrations.
QUESTION 555
The pediatrician has diagnosed tinea capitis in an 8- year-old girl and has placed her on oral
griseofulvin. The nurse should emphasize which of these instructions to the mother and/or
child?
A. Administer oral griseofulvin on an empty stomach for best results.
B. Discontinue drug therapy if food tastes funny.
C. May discontinue medication when the child experiences symptomatic relief.
D. Observe for headaches, dizziness, and anorexia.
Answer: D
Explanation:
(A) Giving the drug with or after meals may allay gastrointestinal discomfort. Giving the
drug with a fatty meal

(ice cream or milk) increases absorption rate.
(B) Griseofulvin may alter taste sensations and thereby decrease the appetite. Monitoring of
food intake is important, and inadequate nutrient intake should be reported to the physician.
(C) The child may experience symptomatic relief after 48–96 hours of therapy. It is important
to stress continuing the drug therapy to prevent relapse (usually about 6 weeks).
(D) The incidence of side effects is low; however, headaches are common. Nausea, vomiting,
diarrhea, and anorexia may occur. Dizziness, although uncommon, should be reported to the
physician.
QUESTION 556
A female client at 10 week’s gestation complains to her physician of slight vaginal bleeding
and mild cramps. On examination, her physician determines that her cervix is closed. The
client is exhibiting signs of:
A. An inevitable abortion
B. A threatened abortion
C. An incomplete abortion
D. A missed abortion
Answer: B
Explanation:
(A) An inevitable abortion includes the signs of cervical dilation and effacement as well as
pain and bleeding.
(B) A threatened abortion is a condition in which intrauterine bleeding occurs early in
pregnancy, the cervix remains undilated, and the uterine contents are not necessarily expelled.
(C) An incomplete abortion occurs when some portions of the products of conception are
expelled from the uterus.
(D) A missed abortion occurs when the embryo dies in utero and is retained in the uterus.
QUESTION 557
A male infant is to be discharged home this morning. Which instruction related to his cord
care should be included in his mother’s discharge teaching plan?
A. Keep the umbilical area moist with Vaseline until the stump falls off.
B. Keep the umbilical area covered at all times with the diaper.
C. Clean the umbilical cord with alcohol at each diaper change.
D. Clean the umbilical cord daily with soap and water during the bath.

Answer: C
Explanation:
(A) The umbilical area should be kept dry for healing to occur. Moisture is conducive to
bacterial growth and therefore could lead to infection at the site.
(B) The diaper should be folded below the cord to allow the cord stump to be exposed to the
air for healing.
(C) The umbilical cord should be swabbed with alcohol at each diaper change to remove
urine and stool and to facilitate the desiccation process through drying.
(D) Soap and water should not be used to clean the umbilical area because the area could
retain moisture, thus making it susceptible to bacterial growth and infection.
QUESTION 558
A male client has heart failure. He has been instructed to gradually increase his activities.
Which signs and symptoms of worsening heart failure should the nurse tell him to watch for
that would indicate a need for him to lower his activity level?
A. Pain in his legs when he walks
B. Thirst, weight loss, and polyuria
C. Drowsiness and lethargy after his activities
D. Weight gain, edema in his lower extremities, and shortness of breath
Answer: D
Explanation:
(A) Pain in the legs could be indicative of doing too much too quickly, but not of worsening
heart failure. The client should be cautioned to increase his activities slowly.
(B) Thirst, weight loss, and frequent urination are not indicative of heart failure. The client
should report these symptoms to his physician.
(C) Drowsiness and lethargy are not indicative of worsening heart failure. The client should
report these symptoms to his physician.
(D) All of these symptoms indicate a worsening cardiac condition possibly associated with
too much activity. The client’s activity level should be evaluated.
QUESTION 559
The physician has ordered that ampicillin 250 mg IV be given over 30 minutes. The
medication is diluted as recommended in 10 mL in the volume control chamber of a set that
has a tubing of 12 mL. Which nursing measure is most accurate considering these facts?

A. Infuse volume at 44 mL/hr.
B. Infuse volume at 22 mL/hr.
C. Infuse volume at 10 mL/hr.
D. Infuse volume at 30 mL/hr.
Answer: A
Explanation:
(A) The volume to be infused should be diluted medication volume added to the volume
control chamber (10 mL) plus the tubing volume (12 mL). The general formula for
calculating IV medications for children is: Rate = Volume to Be Infused X Administration Set
Drop Factor (micro drop: 60 gtts/min) Desired Time to Infuse in Minutes Rate = (10 + 12) 22
X 60 30 = 44 mL/hr.
(B, C, D) These values are incorrect.
QUESTION 560
A 26-year-old client has no children. She has had an abdominal hysterectomy. In the first 24
hours postoperatively, the nurse would be concerned if the client:
A. Cries easily and says she is having abdominal pain
B. Develops a temperature of 102 F
C. Has no bowel sounds
D. Has a urine output of 200 mL for 4 hours
Answer: B
Explanation:
(A) The client may be more tearful than normal due to the stress of the surgery and its
implications for her future life. She would be expected to have pain following surgery.
(B) A temperature of 102°F indicates an infectious process. This is not a normal sequence to
surgery and indicates a need for further assessment.
(C) The client is expected to have no bowel sounds for 24–48 hours after surgery because of
the trauma to the bowel.
(D) Normal urine output is 30 mL/hr. This represents an output of 50 mL/hr, which is greater
than normal.
QUESTION 561
A male client has a history of diverticulosis. He has questions about the foods that he should
eat. His nurse gives him the following information:

A. He should be on a high-fiber diet.
B. He should eat a low-residue diet.
C. He should drink minimal amounts of fluids.
D. He does not need to make any modifications.
Answer: A
Explanation:
(A) Clients with diverticulosis should maintain a high-fiber diet and prevent constipation with
bran or bulk laxatives.
(B) Low residue diets lead to constipation and are contraindicated in clients with
diverticulosis.
(C) Clients with diverticulosis should drink at least eight glasses of water each day to prevent
constipation.
(D) Clients with diverticulosis should modify their diet to include high-fiber foods and bulk
laxatives.
QUESTION 562
A male client is admitted to the psychiatric unit after experiencing severe depression. He
states that he intends to kill himself, but he asks the nurse not to repeat his intentions to other
staff members. Which response demonstrates understanding and appropriate action on the
part of the nurse?
A. “I understand you’re depressed, but killing yourself is not a reasonable option.”
B. “We need to discuss this further, but right now let’s complete these forms.”
C. “Don’t do that, you have so much to live for. You have a wonderful wife and children. The
client in the next room has no one.”
D. “This is very serious. I do not want any harm to come to you. I will have to report this to
the rest of the staff.”
Answer: D
Explanation:
(A) To the client, suicide may be a reasonable action and the only one he can cope with at this
time.
(B) This response indicates to the client that his intention to commit suicide is not important
to the nurse at this time.
(C) The client is so depressed that he is not able to see the positive aspects of his life. At no
time should the nurse discuss another client’s problems in conversation.

(D) This statement tells the client that the nurse recognizes his problem is of a serious nature
and will take all steps necessary to help him.
QUESTION 563
A term neonate has experienced no distress at birth and has an Apgar score of 9. Her mother
has asked to breastfeed her following delivery. Immediately after birth, the neonate was most
susceptible to heat loss. The most appropriate intervention to conserve heat loss and promote
bonding is to:
A. Place her under the radiant warmer
B. Dry her with blankets
C. Place her to her mother’s breast
D. Place her on a heated pad
Answer: C
Explanation:
(A) A radiant warmer maintains an optimal thermal environment by use of a thermal skin
sensor taped to the infant. The warmer limits parental attachment, so, although appropriate, it
is not an intervention that promotes infant attachment.
(B) Warmed blankets prevent heat loss in the neonate by conduction. In addition, tactile
stimuli promote crying and lung expansion. This intervention does not promote attachment,
however.
(C) Skin- to-skin contact is an effective way to conserve heat after delivery and promotes
parental attachment following birth in the healthy term infant. The first period of reactivity
lasts approximately 30 minutes following birth. A strong sucking reflex and an active, awake
newborn characterize this period.
(D) Surfaces of objects warmer than the infant promote overheating by conduction, and
neonatal hyperthermia may result.
QUESTION 564
A 24-year-old male client is admitted with a diagnosis of sickle cell anemia. The nurse
discusses his disease with him and emphasizes the following information:
A. He should monitor his sputum, stools, and urine for signs of bleeding.
B. His daily diet should include a large amount of fluid.
C. He should not be concerned about having to fly on a commuter airplane on a weekly basis.

D. He should not worry about having children because this disease is passed on only by
female carriers.
Answer: B
Explanation:
(A) Bleeding is not a symptom of sickle cell anemia or sickle cell crisis.
(B) Decreased blood viscosity leads to sickling of red blood cells. Increased fluid intake
maintains adequate circulating blood volume and decreases the chance of sickling.
(C) Hypoxia leads to sickling of cells. Flying in no pressurized planes places the client in a
situation of low O2 tension, which can lead to sickling.
(D) Male and female clients with sickle cell disease can pass the trait on to their offspring.
Therefore, this client should receive genetic counseling prior to having children.
QUESTION 565
An alcoholic client who is completing the inpatient segment of a substance abuse program
was placed on disulfiram (Antabuse) drug therapy. What should the nurse include in the
discharge instructions?
A. If disulfiram is taken and alcohol ingested, the client experiences nausea, vomiting and
elevated blood pressure.
B. Disulfiram is most effective when prescribed as late as possible in a recovery program.
C. Disulfiram works on the desensitization principle.
D. The effects of disulfiram can be triggered by alcohol 5 days to 2 weeks after the drug is
discontinued.
Answer: D
Explanation:
(A) When alcohol is ingested with disulfiram therapy, the client experiences nausea,
vomiting, and a potentially serious drop in blood pressure.
(B) Disulfiram is most successful when used early in the recovery process while the
individual makes major lifestyle changes necessary for long-term recovery.
(C) Disulfiram works on the classical conditioning principle.
(D) The effects of disulfiram can be felt when alcohol is ingested 1–2 weeks after disulfiram
is discontinued.

QUESTION 566
As soon as a child has been diagnosed as “hearing impaired”, special education should begin.
Which of the following special education tasks is the most difficult for a severely hearingimpaired child?
A. Auditory training
B. Speech
C. Lip reading
D. Signing
Answer: B
Explanation:
(A) With the slight and mild hard of hearing, auditory training is beneficial.
(B) Speech is the most difficult task because it is learned by visual and auditory stimulation,
imitation, and reinforcement.
(C, D) Lip reading and signing are aimed at establishing communicative skills, but they are
learned more easily by visual stimulation.
QUESTION 567
A female client has just died. Her family is requesting that all nursing staff leave the room.
The family’s religious leader has arrived and is ready to conduct a ceremony for the deceased
in the room, requesting that only family members be present. The nurse assigned to the client
should perform the appropriate nursing action, which might include:
A. Inform the family that it is the hospital’s policy not to conduct religious ceremonies in
client rooms.
B. Refuse to leave the room because the client’s body is entrusted in the nurse’s care until it
can be brought to the morgue.
C. Tell the family that they may conduct their ceremony in the client’s room; however, the
nurse must attend.
D. Respect the client’s family’s wishes.
Answer: D
Explanation:
(A) It is rare that a hospital has a specific policy addressing this particular issue. If the
statement is true, the nurse should show evidence of the policy to the family and suggest
alternatives, such as the hospital chapel.

(B) Refusal to leave the room demonstrates a lack of understanding related to the family’s
need to grieve in their own manner.
(C) The nurse should leave the room and allow the family privacy in their grief.
(D) The family’s wish to conduct a religious ceremony in the client’s room is part of the grief
process. The request is based on specific cultural and religious differences dictating social
customs.
QUESTION 568
A mother brings a 6-month-old infant and a 4-year-old child to the nursing clinic for routine
examination and screening. Which of these plans by the nurse would be most successful?
A. Examine the 4 year old first.
B. Provide time for play and becoming acquainted.
C. Have the mother leave the room with one child, and examine the other child privately.
D. Examine painful areas first to get them “over with”.
Answer: B
Explanation:
(A) The 6 month old should be examined first. If several children will be examined, begin
with the most cooperative and less anxious child to provide modeling.
(B) Providing time for play and getting acquainted minimizes stress and anxiety associated
with assessment of body parts.
(C) Children generally cooperate best when their mother remains with them.
(D) Painful areas are best examined last and will permit maximum accuracy of assessment.
QUESTION 569
A 7-year-old girl has been diagnosed with juvenile arthritis and has been placed on daily
aspirin. Which statement made by the parent indicates a need for further teaching?
A. “My daughter takes her aspirin with her meals.”
B. “Her gums have been bleeding frequently. Maybe she is brushing too hard.”
C. “I give her aspirin on a regular schedule every day.”
D. “One sign of aspirin toxicity can be ringing in the ears.”
Answer: B
Explanation:
(A) Aspirin should not be given on an empty stomach because it is irritating to the mucosa.
(B) Bleeding from decreased clotting capacity may be caused by aspirin toxicity.

(C) A regular schedule of aspirin administration is important to maintain a satisfactory drug
level in the body.
(D) Aspirin toxicity may affect cranial nerve VIII, leading to tinnitus (ringing in the ears).
QUESTION 570
A normal 3-year-old child is suspected of having meningitis. The doctor has ordered a lumbar
puncture. In light of this procedure and developmental characteristics of this age group,
which nursing measure is most appropriate?
A. Emphasize those aspects of the procedure that require cooperation.
B. Tell the child not to cry or yell.
C. Tell the child that he will get a “stick in his back.”
D. Use medical terminology when explaining the procedure to the client.
Answer: A
Explanation:
(A) The nurse should emphasize what is required to elicit cooperation and help to develop a
sense of autonomy.
(B) The child may express discomfort verbally and should be encouraged to express his
feelings.
(C) Selecting nonthreatening words to explain a procedure will prevent misinterpretation.
(D) When explaining the procedure to the parent with the child present, the nurse should use
words that the child can understand to avoid misunderstanding.
QUESTION 571
A 6-year-old girl has been diagnosed with a urinary tract infection secondary to vesicoureteral
reflux. Which statement by her mother indicates a need for further teaching?
A. “I have taught her to wipe from front to back after urinating.”
B. “I make sure she drinks plenty of fluids every day.”
C. “She enjoys wearing nylon panties, but I make her change them every day.”
D. “She tries to empty her bladder completely after she urinates, like I told her.”
Answer: C
Explanation:
(A) Wiping from front to back is wiping from an area of lesser contamination (urethra) to an
area of greater contamination (rectum).
(B) Generous fluid intake reduces the concentration of urine.

(C) Cotton is a natural, absorbent fabric. Nylon often predisposes the client to urinary tract
infections. Dark, warm, moist areas are excellent media for bacterial growth.
(D) With vesicoureteral reflux, urine refluxes into the ureter(s) during voiding and then
returns to the bladder (residual), which becomes a source for future infection.
QUESTION 572
A female client is admitted to the emergency department complaining of severe right-sided
abdominal pain and vaginal spotting. She states that her last menstrual period was about 2
months ago. A positive pregnancy test result and ultrasonography confirm an ectopic
pregnancy. The nurse could best explain to the client that her condition is caused by:
A. Abnormal development of the embryo
B. A distended or ruptured fallopian tube
C. A congenital abnormality of the tube
D. A malfunctioning of the placenta
Answer: B
Explanation:
(A) The embryo itself may develop normally in the first several weeks of an ectopic
pregnancy.
(B) An ectopic pregnancy in the fallopian tube causes severe pain owing to the size of the
growing embryo within the narrow lumen of the tube, causing distention and finally rupture
within the first 12 weeks of pregnancy.
(C) The Fallopian tube may either be normal or contain adhesions caused by a history of
pelvic inflammatory disease or tubal surgeries, neither of which are congenital causes.
(D) An ectopic pregnancy does not involve a dysfunctional placenta, but the implantation of
the blastocyst outside the uterus.
QUESTION 573
The FHR pattern in a laboring client begins to show early decelerations. The nurse would best
respond by:
A. Notifying the physician
B. Changing the client to the left lateral position
C. Continuing to monitor the FHR closely
D. Administering O2 at 8 L/min via face mask
Answer: C

Explanation:
(A) Early decelerations are reassuring and do not warrant notification of the physician.
(B) Because early decelerations is a reassuring pattern, it would not be necessary to change
the client’s position.
(C) Early decelerations warrant the continuation of close FHR monitoring to distinguish them
from more ominous signs.
(D) O2 is not warranted in this situation, but it is warranted in situations involving variable
and/or late decelerations.
QUESTION 574
A female client is anticipating a visit with her parents over the Thanksgiving holidays. She
has recently begun experiencing periods of extreme shortness of breath, which her physician
has labeled as panic attacks. Which of the following statements by the nurse would enhance
therapeutic communication?
A. “Why do you feel this way?”
B. “Tell me about your dislike for your parents.”
C. “Don’t worry, everything will be all right on your visit with your parents.”
D. “Perhaps you and I can discover what produces your anxiety.”
Answer: D
Explanation:
(A) Asking the client to provide an explanation for her feelings is often intimidating.
(B) This response is probing and may make the client feel used and valued only for the
information she can provide.
(C) This underrates the client’s feelings and belittles her concerns. It may cause the client to
stop sharing feelings for fear that they will be ridiculed.
(D) The emphasis is on working with the client. It shows that there is hope for change
through collaboration.
QUESTION 575
A 28-year-old client comes to the clinic for her first prenatal examination. In relating her
obstetrical history, she tells the nurse that she has been pregnant twice before. She had a
“miscarriage” with the first pregnancy after 6 weeks. With the second pregnancy, she
delivered twin girls at 31 week’s gestation. One of the twins was stillborn and the other twin
died at 4 days of age. Using a five-digit system, the nurse records her as being:

A. 2-0-2-1-0
B. 2-2-2-1-2
C. 3-0-1-1-0
D. 2-1-1-0-0
Answer: C
Explanation:
(A) The first digit represents the total number of pregnancies. This client has been pregnant 3
times including this pregnancy. The twin pregnancy counts as only one pregnancy, and
because she delivered prior to 37 week’s gestation, the third digit is recorded as 1.
(B) The first digit represents the total number of pregnancies. This client has been pregnant 3
times including this pregnancy. The second digit represents the total number of full-term
deliveries; she has lost two pregnancies before 37 week’s gestation. At present, she has no
living children, so the fifth digit is noted as 0.
(C) The client is pregnant for the third time, and the first digit reflects the total number of
pregnancies. She has had no full-term deliveries, because she delivered prior to 37 gestational
weeks, so the second digit is recorded as 0. The third digit represents the number of preterm
deliveries, and a twin pregnancy counts as only one delivery. She lost an earlier pregnancy
prior to 20 gestational weeks, and the fourth digit reflects spontaneous or elective abortions.
Lastly, the fifth digit indicates the number of children currently living, and she has no living
children.
(D) She is pregnant for the third time, and the first digit reflects the total number of
pregnancies. In the previous two pregnancies, she delivered prior to 37 gestational weeks,
thus having no full-term deliveries, which is indicated by the second digit. The fourth digit
represents the total number of abortions, spontaneous or elective, and she reported a
spontaneous abortion with her first pregnancy.
QUESTION 576
A murmur has been discovered during the routine physical examination of a 1-year-old child.
The parent is extremely concerned about this diagnosis. Which of the following explanations
by the nurse indicates understanding of this dysfunction?
A. The blood shifts from the right to the left atrium.
B. Surgical closure by suture or patch is recommended before school age.
C. Most atrial septal defects close spontaneously.
D. The child can be treated medically with antibiotics to prevent bacterial endocarditis.

Answer: B
Explanation:
(A) Because the left atrial pressure is greater than right atrial pressure, oxygenated blood
flows from the left to the right atria.
(B) Because of the risk of pulmonary obstructive diseases and congestive heart failure later in
life, surgery is usually performed between age 4 and 6 years, with essentially no operative
mortality or postoperative complications.
(C) Many ventricular septal defects close spontaneously (20–60%) as a result of growth and
proliferation of the muscular septum or formation of a membrane across the opening.
(D) This management is usually recommended with children with mild pulmonary stenosis.
QUESTION 577
Seven days ago, a 45-year-old female client had an ileostomy. She is self-sufficient and well
otherwise. Which of the following long-term objectives would be unrealistic?
A. She should be able to control evacuation of her bowels.
B. She should be able to return to a regular diet.
C. She should be able to resume sexual activity.
D. She should be able to manage her own care.
Answer: A
Explanation:
(A) Because of the location of an ileostomy, the client will not be able to control the
evacuation of her bowels. The ileostomy will drain liquid stool continuously.
(B) The client should be able to return to a normal, well- balanced diet. She should avoid
foods that cause diarrhea or excessive gas production, and she should eat small meals.
(C) The client should be able to resume sexual activity. She will be able to wear a pouch.
(D) The client has no other health or mental problems and should be able to manage her own
ileostomy.
QUESTION 578
A female client is concerned that she is in a “high-risk group” for the development of
acquired immunodeficiency syndrome (AIDS). She wants to know about the advisability of
donating blood. Which of the following responses is correct?
A. “Individuals who donate blood are at risk of getting the AIDS virus. You should not
donate.”

B. “It’s OK for you to donate because the blood bank has a test that is 100% effective.”
C. “You should not donate since it takes time to develop antibodies to the AIDS virus. If you
donate blood before you develop the antibody, you could pass it on in the blood.”
D. “It is not a good idea for you to donate. If you have AIDS, the information is made public
and could destroy your personal life.”
Answer: C
Explanation:
(A) The AIDS virus cannot be transmitted to the donor through the blood donation procedure.
(B) The test for the AIDS virus is not absolutely foolproof; therefore, it is not wise for a
person with known risk factors to donate blood.
(C) It takes time for antibodies to the AIDS virus to develop. An infected individual could
donate contaminated blood without it testing positive for the virus.
(D) For reasons of confidentiality, information about individuals infected with AIDS is not
made public.
QUESTION 579
A 40-year-old client has lived for 8 years with an abusive spouse. She married her husband in
her senior year of high school after becoming pregnant. Shortly after the baby was born, he
began to physically abuse her. She has attempted to leave him several times, but she has
always returned. She is unable to support herself financially, and her husband threatens to kill
her if she leaves him. This time, her husband has beaten her so badly she cannot stop the
bleeding from the gash above her eye. She admits her husband caused her injury. In assessing
a person after experiencing spousal abuse, which need has the highest priority?
A. Assess the level of anxiety, coping responses, and support systems.
B. Assess the history of physical abuse.
C. Assess suicide potential.
D. Assess drug and alcohol use.
Answer: C
Explanation:
(A) Assessing the level of anxiety, coping responses, and support systems is very important,
but not of highest priority at this time.
(B) A history of physical abuse is an important part of assessment. The nurses must also
always ask if there is abuse of the children.

(C) Although all of these answers are very important in assessment, the highest priority is
assessment of suicide potential, because this could cause the greatest harm to the client.
Feeling there is no other way out, abused spouses may consider suicide.
(D) The spouse may be self- medicating herself with alcohol or drugs to escape an awful
situation. The abuser may also be abusing drugs or alcohol. If this is so, the nurse should
encourage the spouse to seek counseling and not to return to the home.
QUESTION 580
In an interview for suspected child abuse, the child’s mother openly discusses her feelings.
She feels her husband is too aggressive in disciplining their child. The child’s father states,
“Being a school custodian”, I see kids every day that are bad because they did not get enough
discipline at home. That will not happen to our child. Based on this remark, the nurse would
make the following nursing diagnosis:
A. Fear related to retaliation by the father
B. Actual injury related to poor impulse control by the father
C. Ineffective coping
D. Altered family process related to physical abuse
Answer: D
Explanation:
(A) There is no evidence of fear as the child is unable to communicate.
(B) There is actual injury, but the parents have not yet admitted causing the child’s injuries.
(C) This diagnosis is incomplete. There is no specific ineffective coping behavior identified in
this nursing diagnosis.
(D) Altered family process best describes the family dynamics in this situation. The parents
have admitted severe disciplinary action.
QUESTION 581
A male client is being treated in the burn unit for third-degree burns on his head, neck, and
upper chest received in the last 24 hours. The nurse is evaluating the effectiveness of fluid
resuscitation. Which of the following indicates effective fluid balance?
A. His weight increases from 165 to 175 lb.
B. His urine output is equal to his total fluid intake.
C. His urine output has been 35 mL/hr for the past 12 hours.
D. His blood pressure is 94/62.

Answer: C
Explanation:
(A) A weight gain of 10 lb represents a state of overhydration.
(B) He is losing fluids through insensible losses; a urine output equal to his intake indicates
that he is receiving too little fluids.
(C) A urine output greater than his intake indicates that he is receiving adequate fluid
resuscitation to account for urinary and insensible losses.
(D) A blood pressure of 94/62 indicates a state of underhydration and inadequate circulatory
volume.
QUESTION 582
A male client is undergoing cardiac tests. He has been instructed to wear a Holter monitor.
The nurse knows she has included the appropriate information in her teaching when the client
tells her:
A. “He should remove the electrodes for bathing.”
B. “Damage to his heart muscle will be recorded by the monitor.”
C. “He is to keep a record of everything he does during the day.”
D. “He is to refrain from activities that cause chest pain.”
Answer: C
Explanation:
(A) The client should leave the electrodes in place during the entire time the test is ordered.
He should not even remove the electrodes for bathing.
(B) The Holter monitor will record cardiac electrical activity but will not record damage to
his myocardium.
(C) The client should keep a record of all of his activities so the physician can correlate the
ECG findings with his activities.
(D) The client should continue doing his regular activities. The purpose of the Holter monitor
is to record heart activity during routine activities.
QUESTION 583
A 3-year-old child has had symptoms of influenza including fever, productive cough, nausea,
vomiting, and sore throat for the past several days. In caring for a young child with symptoms
of influenza, the mother must be cautioned about:
A. Giving aspirin and bismuth subsalicylate (Pepto-Bismol) to treat the symptoms

B. Giving clear liquids too soon
C. Allowing the child to come in contact with other children for 3 days
D. The possibility of pneumonia as a complication
Answer: A
Explanation:
(A) Aspirin should never be given to children with influenza because of the possibility of
causing Reye’s syndrome. Pepto- Bismol is also classified as a salicylate and should be
avoided.
(B) Depending on the severity of symptoms, the child may be receiving IV therapy or clear
liquids.
(C) The disease has a 1–3 day incubation period and affected children are most infectious 24
hours before and after the onset of symptoms.
(D) Although viral pneumonia can be a complication of influenza, this would not be an initial
priority.
QUESTION 584
A male client has burns over 90% of his body after an automobile accident resulting in a fire.
He was trapped inside the auto and pulled out by a bystander. After several months in the
hospital and over 20 surgeries, discharge planning has begun. Throughout his hospitalization
the nursing staff has been aware of psychological changes the client faces after burns over a
large portion of his body resulting in disfigurement. The nursing staff can best foster the
client’s self-esteem by:
A. Adhering to a strict schedule of diet, exercise, and wound care
B. Allowing him to go to physical therapy for whirlpool treatment when other clients were
not in physical therapy
C. Following a standardized plan of care for burn clients formulated by a world-renowned
burn center
D. Allowing him to plan, assist in, and perform his own care whenever possible
Answer: D
Explanation:
(A) A regimented schedule, allowing no flexibility, will not foster the client’s self-esteem.
(B) Isolating the client may only enhance his feelings of social isolation due to his
disfigurement.
(C) Standardized care plans must be personalized and adapted to each client’s situation.

(D) Allowing the client control over his care will foster his self-esteem and prepare him for
life outside of the hospital.
QUESTION 585
A female client decides on hemodialysis. She has an internal vascular access device placed.
To ensure patency of the device, the nurse must:
A. Assess the site for leakage of blood or fluids
B. Auscultate the site for a bruit
C. Assess the site for bruising or hematoma
D. Inspect the site for color, warmth, and sensation
Answer: B
Explanation:
(A) This is an internal device. Assessment of the site should include assessing for swelling,
pain, warmth, and discoloration. This measure does not assess patency.
(B) The presence of a bruit indicates good blood flow through the device.
(C) The nurse should inspect the site for bruising or hematoma; however, this measure does
not assure patency of the device.
(D) The nurse should inspect the vascular access site frequently for signs of infection.
However, this does not assure patency.
QUESTION 586
A female client at 30 week’s gestation is brought into the emergency department after falling
down a flight of stairs. On examination, the physician notes a rigid, board like abdomen; FHR
in the 160s; and stable vital signs. Considering possible abdominal trauma, which obstetric
emergency must be anticipated?
A. Abruptio placentae
B. Ectopic pregnancy
C. Massive uterine rupture
D. Placenta previa
Answer: A
Explanation:
(A) Abruptio placentae, the complete or partial separation of the placenta from the uterine
wall, can be caused by external trauma. When hemorrhage is concealed, one sign is a rapid
increase in uterine size with rigidity.

(B) Ectopic pregnancy occurs when the embryo implants itself outside the uterine cavity.
(C) Massive uterine rupture occurs during labor when the uterine contents are extruded
through the uterine wall. It is usually due to weakness from a pre-existing uterine scar and
trauma from instruments or an obstetrical intervention.
(D) Placenta previa is the condition in which the placenta is implanted in the lower uterine
segment and either completely or partially covers the cervical os.
QUESTION 587
A client has just received an epidural block. She is laboring on her right side. The nurse notes
that her blood pressure has dropped from 132/68 to 78/42 mm Hg. The nurse’s first action
would be to:
A. Call the physician immediately and give dopamine IM
B. Turn her on her left side and recheck her blood pressure in 5 minutes
C. Administer oxytocin (Pitocin) immediately and increase the rate of IV fluids
D. Increase the rate of IV fluids and start O2 by mask
Answer: D
Explanation:
(A) Nursing measures to support fetal oxygenation and promote maternal blood pressure
would precede calling the physician.
(B) Systolic pressures below 100 mm Hg or a reduction in the systolic pressure of 30%
necessitate treatment. Assessing the blood pressure in 5 minutes may allow for further fetal
and/or maternal compromise. Turning the client on her left side will promote uteroplacental
perfusion and is appropriate.
(C) Oxytocin (Pitocin) increases the strength of uterine contractions and may cause maternal
hypotension; thus it is an inappropriate drug for use in this clinical situation. IV fluids would
be increased to expand the circulating blood volume and promote increased blood pressure.
(D) Turning the mother to her left lateral side promotes uteroplacental perfusion. IV fluids are
administered to increase the circulating blood volume, and O2 is administered to promote
fetal oxygenation and decrease the nausea accompanying the hypotension.
QUESTION 588
A female client at 36 week’s gestation has been treated successfully for premature labor for 4
weeks. She has begun having uterine contractions today and has been admitted to the labor

and delivery suite. Her amniocentesis results reveal a lecithin/sphingomyelin (L/S) ratio of 2
and positive phosphatidylglycerol (PG). These lab values indicate:
A. Placental maturity
B. Suspected chronic asphyxia
C. Cord compression
D. Fetal lung maturity
Answer: D
Explanation:
(A) Placental maturity is assessed by a biophysical profile.
(B) L/S ratio and presence of phosphatidylglycerol are not used to determine fetal asphyxia. A
biophysical profile score of 6 may indicate this condition.
(C) Cord compression is not reflected by the L/S ratio or presence of phosphatidylglycerol.
Variable decelerations observed through electronic fetal monitoring could reflect umbilical
cord compression.
(D) An L/S ratio 2 and the presence of phosphatidylglycerol in amniotic fluid indicate fetal
lung maturity.
QUESTION 589
The postpartum nurse should include which of the following instructions to breast-feeding
mothers?
A. Limit feeding times for several days to avoid nipple soreness.
B. Wash the nipples with soap and water before and after each feeding.
C. Daily caloric intake should be increased by 500 cal.
D. Breast milk is totally digestible by the baby because it contains lactose.
Answer: C
Explanation:
(A) Limiting initial feeding times will only delay nipple soreness as well as the establishment
of the letdown reflex, thus encouraging engorgement from clogged ducts and ductules.
(B) Soap should be avoided because it may be excessively drying, predisposing nipples to
cracking.
(C) For optimal milk production, an additional 500 kcal over maintenance levels are needed
daily.
(D) Lipase, not lactose, emulsifies the fat in breast milk, making it almost totally digestible by
infants.

QUESTION 590
A primigravida with a blood type A negative is at 28 week’s gestation. Today her physician
has ordered a RhoGAM injection. Which statement by the client demonstrates that more
teaching is needed related to this therapy?
A. “I’m getting this shot so that my baby won’t develop antibodies against my blood, right?”
B. “I understand that if my baby is Rh positive I’ll be getting another one of these injections.”
C. “This shot should help to protect me in future pregnancies if this baby is Rh positive, like
my husband.”
D. “This shot will prevent me from becoming sensitized to Rh-positive blood.”
Answer: A
Explanation:
(A) RhoGAM is given to Rh-negative mothers to prevent the maternal Rh immune response
to fetal Rh-positive antigens.
(B) If the infant is Rh positive, the mother will receive another dose postdelivery to prevent
maternal sensitization.
(C) Prevention of maternal sensitization will protect future pregnancies because the mother’s
blood will be free of antibodies against her fetus.
(D) RhoGAM prevents maternal sensitization to Rh-positive blood.
QUESTION 591
A client has been instructed in how to take her nitroglycerin tablets. The nurse giving her
instructions knows the client understands the information when she tells her:
A. “I should contact my physician if I have headaches after I take this medicine.”
B. “I should keep the tablets in the refrigerator.”
C. “I should call the doctor if three doses of the medicine do not relieve my pain.”
D. “I should take these with water but not with milk.”
Answer: C
Explanation:
(A) Headaches may occur after taking nitroglycerin because of vasodilation.
(B) The tablets do not need to be refrigerated. The client should carry them with her.
(C) The client should contact the physician if repeated doses of nitroglycerin do not relieve
the discomfort.
(D) Nitroglycerin tablets should be dissolved under the tongue, not swallowed.

QUESTION 592
At her monthly prenatal visit, a client reports experiencing heartburn. Which nursing measure
should be included in her plan of care to help alleviate it?
A. Restrict fluid intake.
B. Use Alka-Seltzer as necessary.
C. Eat small, frequent bland meals.
D. Lie down after eating.
Answer: C
Explanation:
(A) At least eight glasses of fluid per day are encouraged to help dilute stomach contents,
thereby decreasing irritation.
(B) Alka Seltzer contains aspirin, which is irritating to gastric mucosa, and therefore should
be avoided.
(C) Small, frequent bland meals help to decrease gastric pressure and to prevent reflux.
(D) Lying down after meals may cause gastric reflux and prevents optimal gastric emptying.
QUESTION 593
A 14-year-old teenager is demonstrating behavior indicative of an obsessive-compulsive
disorder. She is obsessed with her appearance. She will not leave her room until her hair,
clothes, and makeup are perfect. She always dresses immaculately. Recently, she expressed
disgust over her appearance after she gained 5 lb. After observing a marked weight loss over
a 2-week period, her mother suspects that she is experiencing bulimia. She eats everything on
her plate, then runs to the bathroom. In interviewing the teenager, she discusses in great detail
all of the events leading to her bulimia, but not her feelings. What defense mechanism is she
using?
A. Dissociation
B. Intellectualization
C. Rationalization
D. Displacement
Answer: B
Explanation:
(A) Dissociation is separating a group of mental processes from consciousness or identity,
such as multiple personalities. That is not evident in this situation.

(B) Intellectualization is excessive use of reasoning, logic, or words usually without
experiencing associated feelings. This is the defense mechanism that this client is using.
(C) Rationalization is giving a socially acceptable reason for behavior rather than the actual
reason. She is discussing events, not reasons.
(D) Displacement is a shift of emotion associated with an anxiety-producing person, object,
or situation to a less threatening object.
QUESTION 594
As a nurse in the emergency room, you receive an outside call from an elderly woman who
states she has just been raped. She states, “I know” I must come to the hospital, but what do I
do next? You advise her to call the police, then come to the hospital emergency room. What
action by the nurse would indicate an understanding of the examination process once the
victim enters the emergency room?
A. Inform the victim not to wash, change clothes, douche, brush teeth, or eat or drink
anything.
B. Inform the victim to bring insurance information with her to the hospital so she can be
properly cared for.
C. Phone a rape counselor to begin working with the victim as soon as she enters the hospital.
D. Do not leave the victim alone to collect her thoughts.
Answer: A
Explanation:
(A) Providing the victim with these instructions will aid in the determination of physical
evidence of rape. Victims frequently feel “dirty after rape”, and their first instinct is to take
care of personal hygiene before facing anyone.
(B) This action is of lesser importance at this time.
(C) Although this is a nursing measure appropriate in this situation, contacting a counselor
can be done once the victim enters the hospital. Frequently victims call but do not follow up
with the visit.
(D) Once the victim enters the emergency room, it is important not to leave her alone.
QUESTION 595
A 15-year-old child is admitted to the pediatric unit with a diagnosis of thalassemia. Which of
the following would be included in educating the mother and child as part of discharge
planning?

A. Give oral iron medication every day.
B. Have the child’s blood pressure monitored every week.
C. Know the signs and symptoms of iron overload.
D. Keep exercise at a minimum to reduce stress.
Answer: C
Explanation:
(A) Oral iron supplements are contraindicated in thalassemia.
(B) Although heart failure may be an end result of this disease, this action is unnecessary.
(C) Iron overload is a potential complication of frequent blood transfusions of children with
thalassemia.
(D) Children should be encouraged to pursue activities related to their exercise tolerance.
QUESTION 596
A female client was recently diagnosed with gastric cancer. She entered the hospital and had a
total gastrectomy with esophagojejunostomy. Her postoperative recovery was uneventful. On
conducting discharge teaching, the nurse discusses changes in bodily function and lifestyle
changes with the client. In order to prevent pernicious anemia, the nurse stresses that the
client must:
A. Receive monthly blood transfusions
B. Increase the amount of iron in her diet
C. Eat small quantities several times daily until she is able to tolerate food in moderate
portions
D. Understand the need for Vitamin B12 replacement therapy
Answer: D
Explanation:
(A) Monthly blood transfusions are not indicated post gastrectomy.
(B) Increasing iron in the client’s diet may cause irritation and will not alleviate pernicious
anemia.
(C) It may be necessary that the client eat small meals several times per day, but this measure
has no relevance to prevention of pernicious anemia.
(D) Pernicious anemia is caused by lack of Vitamin B12, and replacement therapy will be
necessary because the client’s stomach has been removed.

QUESTION 597
An 8-year-old boy has been diagnosed with hemophilia. Which of the following diagnostic
blood studies is characteristically abnormal in this disorder?
A. Partial thromboplastin time
B. Platelet count
C. Complete blood count
D. Bleeding time
Answer: A
Explanation:
(A) Partial thromboplastic time measures activity of thromboplastin, which depends on the
intrinsic clotting factors deficient in children who are hemophiliacs.
(B) Platelet counts are normal in hemophilia.
(C) Hemophilia does not affect the complete blood count.
(D) Bleeding times are normal in hemophiliacs. They measure the time interval for the
bleeding from small superficial wounds to cease.
QUESTION 598
A 44-year-old female client is receiving external radiation to her scapula for metastasis of
breast cancer. Teaching related to skin care for the client would include which of the
following?
A. Teach her to completely clean the skin to remove all ointments and markings after each
treatment.
B. Teach her to cover broken skin in the treated area with a medicated ointment.
C. Encourage her to wear a tight-fitting vest to support her scapula.
D. Encourage her to avoid direct sunlight on the area being treated.
Answer: D
Explanation:
(A) The skin in a treatment area should be rinsed with water and patted dry. Markings should
be left intact, and the skin should not be scrubbed.
(B) Clients should avoid putting any creams or lotions on the treated. This could interfere
with treatment.
(C) Radiation therapy clients should wear loose-fitting clothes and avoid tight, irritating
fabrics.

(D) The area of skin being treated is sensitive to sunlight, and the client should take care to
prevent sun damage by avoiding direct sunlight and covering the area when she is in the sun.
QUESTION 599
A client has renal failure. Today’s lab values indicate he has an elevated serum potassium.
What additional priority information does the nurse need to obtain?
A. Evaluation of his level of consciousness
B. Evaluation of an electrocardiogram
C. Measurement of his urine output for the past 8 hours
D. Serum potassium lab values for the last several days
Answer: B
Explanation:
(A) The level of consciousness is not affected by elevated potassium levels.
(B) An electrocardiogram (EKG) can tell the nurse whether this client is experiencing any
cardiac dysfunction or arrhythmias related to the elevated potassium level.
(C) Measurement of the urine output is not a priority nursing action at this time.
(D) The client’s serum potassium values for the past several days may provide information
about his renal function, but they are not a priority at this time.
QUESTION 600
At 12 hours postvaginal delivery, a female client is without complications. Which of the
following assessment findings would warrant further nursing interventions?
A. Apical pulse of 52 bpm
B. Uterine fundus palpable left of midline
C. No bowel movement since delivery
D. Oral temperature of 100.4°F
Answer: B
Explanation:
(A) Bradycardia of 50–70 bpm may be considered normal postpartally because the heart
compensates for the decreased resistance in the pelvis.
(B) The uterus is displaced from the midline by a full bladder. This condition could lead to a
boggy uterus and increased risk of postpartal hemorrhage; therefore, the bladder should be
kept empty.
(C) Re-establishment of normal bowel function is delayed into the first postpartum week.

(D) A postpartum woman’s oral temperature may go as high as 100.4°F within 24 hours of
delivery resulting from muscular exertion, dehydration, and hormonal changes.

Document Details

Related Documents

person
Harper Mitchell View profile
Close

Send listing report

highlight_off

You already reported this listing

The report is private and won't be shared with the owner

rotate_right
Close
rotate_right
Close

Send Message

image
Close

My favorites

image
Close

Application Form

image
Notifications visibility rotate_right Clear all Close close
image
image
arrow_left
arrow_right